nevenka tadic pdm - skripta

80

Click here to load reader

Upload: jana

Post on 29-Jun-2015

3.543 views

Category:

Documents


32 download

TRANSCRIPT

Page 1: NEVENKA TADIC PDM - skripta

NEVENKA TADIC PDM

1. Kriterijum normalnosti1) Procena u odnosu na nazadovanje2) U odnosu na onoga ko procenjuje i donosi sud3) U odnosu na osobu koju prrocenjujemo4) U odnosu na merila statističkog proseka5) U odnosu na prilagođenost i neprilagođenost

2. Normalno i patološko kod dece i mladih u poremećaju ponašanjaPotrebno je razlikovati stvaralačko prilagođavanje od nestvaralačkog, odnosno od potčinjavanja. Zato

dobro ili loše prilagođavanje mora biti određeno, kako u odnosu na prilagođavanje prema sebi-autoplastično, tako i u odnosu na druge-autoplastično, da bi se procenilo njegovo pojedinačno psihološko i društveno značenje i vrednost.

3. Normalno i patološko u odnosu na prilagođavanjePrilagođenost - normalno - znači odsustvo sukoba, kako unutar sklopova ličnosti, tako i između osobe i društva.Neprilagođeno - patološko - se obično traži u pojedinačnoj osobi, u njenoj užoj društvenoj sredini, a manje u širem društvu.

4. Dva najglobalnija kriterijuma za razlikovanje normalno - patološkoPatološko bi predstavljalo nesposobnost mlade osobe da obzirom na uzrast: Zadovoljava svoje osnovne biološke, osećajne, saznajne i društvene potrebe Da nadzire svoju agresivnost, depresivnost i strepnju Da izdrži osujećenja Da se trajno ili barem najčešće ponaša na društveno prihvatljiv način i ne oštećuje druge Da ostvaruje dobre i trajne društvene odnose sa drugima Da ostvaruje pozitivne i stvarne društvene ciljeve Da se poistovećuje sa etičkim vrednostima Da ostvari svoje biološke datostiVidljiva jačina simptoma mora biti procenjena u odnosu na snage sistema ličnosti da izdrže unutrašnji ili spoljni sukob koji je pojavu izazvao.

5. Težišne godine U elektrokortikalnom sazrevanju. To su godine - prva, treća, šesta, jedanaesta, u kojima se bioelektrične promene završavaju. Ove etape odgovaraju razvojnim fazama - Prelasku od malog deteta ka predškolskom - 3-6 god. Školskom 7.god. Početku puberteta 11-12 god.

6. Nasledne karakteristike ličnostiNaslednost – heritabilnostVeća nasledljivost je za intelektualne sposobnosti → govorne, saznajne, prostorne i asocijativne.Nasledljivost crtra ličnosti se mnogo teže procenjuje – zbog uticaja spolj. sredine.Obeležja ličnosti naslednog porekla su:

1) Energetski kapacitet, 2) Osećajnost, 3)Sklonost ka strepnji,4) Impulsivnost, 5)Agrsivnost

7. K-ke vulnerabilne dece

1

Page 2: NEVENKA TADIC PDM - skripta

1) Niže energetske mogućnosti2) Manje krepka i delatna deca3) Manje zainteresovana i poverljiva4) Odnos sa roditeljima je slabiji

Povišena povredljivost CNS-a koja je najveća u prva 3, a zatim do 6. meseca embrionog života, za vreme stvaranja neuroblasta, iz kojih se stvaraju neuroni.

8. 1Po Hartmanu koje f-ja učestvuju u procesu prilagođavanjaHartman - govori o “prilagodljivosti” deteta i o autonomnim funkcijama ja koje postoje i pre iskustva sa stvarnošću i koje predstavljaju delove ja bez sukoba. Značajan činilac u povišenoj povredljivosti je nemogućnost da se odgovori na draži i da se nadziru rasterećenja.

9. Koji su po Frojdu pokazatelji jačine EgaJak Ego dobro kontroliše unutrašnju realnost i uspešno izlazi na kraj sa slepim prohtevima ličnosti, uz istovremeno jasno i pouzdano uviđanje spoljnih mogućnosti.Meh. odbrane jakog Ega su takođe, postojani i uspešno funkcionišu.

10. Da li se nagoni svesno opažajuNagoni nemaju kvalitet da bi bili opaženi od svesti. Tek predstava o nagonu, npr. vidna slika/slušni utisak mogu postati svesni. Najpoznatije osvešćenje se postiže kroz reči. Mentalni sadržaj koji je povezan sa govornim ili čulnim pamćenjem dobija potrebna svojstva da bi mogao dopreti do svesti, a drugi uslov da misao dopre do svesti je da bude katektirana.

11. Na kom nivou psihosex. razvoja se javlja fiksacija kod konverzijafalusni nivo

12. Fiksacija falusne fazeFiksacija (zadržavanje) falusnih svojstava u odraslom dobu ometa put ka konačnoj genitalizaciji i dalje mogućnost za razvoj neuroza, u prvom redu konverzivno-histerične neuroze.

13. Objasni pojam preobjekta Objekt predstavlja osobu, deo tela, predmet i sve ono sto sluzi ulaganju nagonske energije i zadovoljenju

nagonskog cilja. Može biti spoljašnji, unutrašnji i deo vlastitog tela. U preobjektnom stadijumu dete živi u svetu bez objekta. U toj fazi dete ne razlikuje ja od ne-Ja a grudi

majke opaža i doživljava kao deo samog sebe. Beba ne razlikuje objekte, ni majku, mada ona odgovara na njene signale. Ona odašilje znake koji postižu

svoj cilj. Ona ne opaža objekt, ali doživljava rasterećenje od neprijatnosti, npr. od gladi onda, kada je odgovoreno na njen znak-plač.

To je uslov da se prepozna objekt i uspostavi odnos sa objektom.

Špic - preobjektni stadijum - Psihoanaliza – stadijum primarnog narcizmaŠpic - stadijum preteče objekta - Psihoanaliza – anaklitički stadijumŠpic - objektni stadijum - Psihoanaliza - oralni stadijum

- Psihoanalitički stadijum preteče objekta-.Preobjektni stadijum

2

Page 3: NEVENKA TADIC PDM - skripta

-Stadijum primarnog narcizma

Preobjektni stadijum (po psihoanalizi stadijum primarnog narcizma). Novorođenče živi u svetu bez objekta, ne razlikuje se od ne-ja, grudi majke doživljava kao deo sebe. Prelazi iz stanja zadovoljstva u stanje nezadovoljstva i obrnuto. Krajem drugog meseca ljudi zauzimaju neko određeno mesto u okolini odojčeta. Beba odašilje znake, poruke koji postižu svoj cilj. Ono ne doživljava objekt nego doživljava rasterećenje od neprijatnosti kad se odgovori na poruku. To je uslov da se prepozna objekt i uspostavu određeni stadijum.

Stadijum preteče objekta (anaklitički po psihoanalizi) od 8-12 nedelja. Opaža lice majke spreda, čelo, oči, nos.

Treći mesec – prati kretanje i smeši se. Uči prvi signal oko 6. nedelje u tragovima sećanju. Zato se govori o preteči objekta, koji povezuje lice majke sa dodirom preko usta i zadovoljenjem potreba tokom maženja. To vodi dete od primarnog narcizma do objektnog stadijuma. Ovo pored društvenog osmeha je tačka organizator, to je trenutak izlaženja odnosa sa sobom u odnos sa okolinom i prelomna je tačka u organizovanju budućeg razvoja na višoj ravni. – Špic

14. Anaklitički strahU periodu između 6 i 8, do 10 meseci dete ulazi u objektni stadijum.Objekat, majka-neka druga osoba se sada doživljava kao celina.majka postaje psihička majka, koja ispunjava dete radošću i onda kad ono nema bioloških.potreba - za hranom...U ovom periodu prekid odnosa sa bliskom i uspostavljanje odnosa sa stranom osobom ispunjava dete strepnjom, opisanu kao anaklitički strah osmog meseca.Pojava anaklitičkog straha je dokaz da majka postaje posebna osoba jedna i jedina u tom razdoblju koja ima svoju posebnu f-ju i ne može biti pomešana sa drugim osobama.

15. Koje tri faze prolazi dete u razvoju identiteta (po Maleru)Za M. Maler strepnja je izazvana prekidom simbiotskog odnosa između deteta i majke i doživljavanjem nestajanja, a zatim procesima odvajanja i individuacije koji se odvijaju u tri faze:

Približavanje,Individuacija,Separacija.

16. Primarni i sekundarni meh. odbranePrimarni:

1) Pounutrenje (čiji je fiziološki osnov unošenje u telo spoljašnjeg objekta),2) Projekcija,3) Cepanje,4) Pomeranje,5) Poricanje,6) Idealizacija,7) Potiskivanje,8) Projektivna identifikacija,9) Izdvajanje,10) Intelektualizacija,11) Randomizacija, ???12) Sublimacija.

Sekundarni: (paranoidno - šizoidni)1) Projekcija,

3

Page 4: NEVENKA TADIC PDM - skripta

2) POunutrenje,3) Cepanje, a u depresivnoj se pridružuju:

Idealizacija,PoricanjeProjektivna identifikacija,Manične odbrane,prisilne odbrane i potiskivanje koje je izgubilo posebno značenje koje je davala psihoanaliza.

17. Projektivna identifikacija – ciljevi (Bion-majka kontejner)Melani Klajn opisujući depresivnu poziciju, M. Klajn uvodi nov mehanizam odbrane -projektivna identifikacija – koja počiva na shvatanju o fantazmima, projekciji, pounutrenju, cepanju i zavisti. Zahvaljujući cepanju u Ja veliki deo mržnje prema delovima vlastite ličnosti upravljen je i projektovan u druge. Ja deteta je zaplašeno jer objekat-majka koju doživljava svemoćnom kao i sebe, može biti opasna, preplavljena nasiljem i mržnjom koje joj je ono projektovalo. Majka svoju mržnju zbog želje za odmazdom može ubaciti, projektovati nazad u dete. Ovaj deo majke dete u fantazmima doživljava kao progonitelja. Pošto je progoniteljski deo majke ponovo pounutren on deluje iznutra, u Ja deteta. Ovo ponovno pounutrenje objekta koji su projektovani napolje u druge osobe, uslov su za uspostavljanje narcističkog odnosa sa objektom. Putem projekcije Ja se oslobađa loših unutrašnjih delova. Objekat u koji je dete projektovalo loše delove postaje progonitelj koji sa svoje strane sada projektuje u dete loše sadržaje. Dete projektuje u druge i svoje dobre delove, Ideaj Ja, tada teži da izbegne odvajanje od objekta ili da ga idealizuje. Zahvaljujući ovim stalnim projekcijama i pounutrenjima koji se odvijaju u fantazmima i zahvaljujući fantazmima o svojoj svemoći dete ima doživljaj da poseduje spoljašnji objekat. Objekat postaje predstavnik ja.

Bion – odojče projektuje u grudi, zatim u celu majku deo svojih sadržaja. Ako je majka sposobna da “sanjara” sa svojim detetom, da prepozna šta se u njemu događa, sadržaj koji dolazi iz odojčeta podleže obradi. Majka postaje kontejner sadržaja koji je dete u nju projektovalo i misli o njima. Ovako promišljen, izmenjen, ublažen i oslobođen neprijateljski sadržaj dete može da pounutri ponovo. Ovo je put ka jačanju ja i stvaranja aparata za mišljenje deteta.

Ciljevi projektivne identifikacije:Da se otarase svog lošeg delaDa poseduje objekt u koji je ubacio svoj deoDa izbegne odvajanje od deteta

18. Projektivna identifikacija kod adolescenataPostoji, kod psihotičnih čija je ličnost prepunjena zavišću i pohlepom. Kako su strepnja i neprijateljstvo jaki, projektovani delovi se cepaju na mnogo komada. Projektovanje ovih komadića u spoljašnji objekt stvara utisak da su i objekti koji primaju raspadnuti. Objekat je sad ispunjen neprijateljskim delovima. Svet se doživljava kao da je nastanjen zastrašujućim i neobičnim bizarnim objektima. Delići ličnosti i objekti se ujedinjuju u uzajamno-razornu vezu, značaj u nastanku psihoza i border-lajna.

19. Mehanizmi odbrane identifikacija sa agresorom i projektivna identifikacijaI. Mehanizam odbrane projektivne identifikacije – M. Klajn je uvela projektivnu i. Kao nov mehanizam

odbrane. Počiva na njenim shvatanjima o:Fantazmima,Projekciji,Pounutrenju,Cepanju i Zavisti.

II. Mehanizam odbrane identifikacije sa agresorom – A. Frojd :Poistovećenje sa moćnim, dobrim,

4

Page 5: NEVENKA TADIC PDM - skripta

Pountrenje sa neprijateljskim i nedruštvenim porukama koje nesvesno upućuju roditelju.

20. . Imitacija (kada se javlja)Pored poistovećenja koje pomaže jasnijem odvajanju sebe od drugih i sticanju identiteta, vrlo je bitna i imitacija (oponašanje) koje se više ne svrstava u nesvesne mehanizme odbrane.Javljaju se u drugoj godini života.

21. Razlika primarne i sekundarne identifikacije na kom uzrastu se javljaPočetno, prvobitno poistovećivanje (primarna identifikacija) deluje u prvim mesecima i godinama života (do 3. g.), najosetljivijoj i uticajima najpodložnijoj razvojnoj fazi. Dete pounutri i prvobitno se poistoveti sa čulnim utiscima (dodirnim, zvučnim, svetlosnim, mirisnim...) koje mu upućuju najbliže osobe. Sveukupna davanja roditelja su prvo pounutrena, a zatim, ili istovremeno, poistovećena sa sobom. Ne treba zaboraviti da je ovo faza razvoja u kojoj ne postoji jasna granica između JA i NE-JA, unutrašnjeg i spoljašnjeg.U rvoj fazi sekundarne ili strukturirajuće identifikacije, od 3 -5 god. života, nastavlja se uobličavanje JA i Nad JA. Dete je u to vreme manje zavisnije od drugih objekata nego u fazi primarne identifikacije. Donekle je uspostavljen unutrašnji nadzor nad unutrašnjim i spoljnim podsticajima i dete delimično ili sasvim usvaja osećanja, ponašanja, govor, polni identitet...Nakon puberteta JA, snabdeveno vlastitim iskustvima, manje se podvrgava spoljnim slikama i uzorima, a više se oslanja na svoja vlastita iskustva, sisteme vrednosti i delom izgrađen JA ideal.Ako su učvršćenja u prethodnim fazama razvoja jaka, omladinac teže dostiže ili i ne dostiže ravan nezavisnosti i poistovećivanja i ostaje izrazito zavisan od roditeljskih slika.

22. Prednosti i nedostaci identifikacije sa moćnim i neodoljivim roditeljimaPrednosti:

Vodi dete do osećanja vlastite moći, što je uobičajeno i poželjno,Nedostaci:

Prejako poistoveć. Kao zaštita od gubitkasvemoćnog roditelja vodi dete ka pogrešnoj slici u jedinstvu sa roditeljem i u strah od svakog odvajanja, prekida odnosa, osamostaljivanja i stvaranja svoje osobenosti. U slučaju gubitka ovakvog odnosa dete ispoljava : duboku strepnju, osećanje bezpomoćnosti, depresivne sadržaje... Nazadovanje izazvano strepnjom odvajanja može biti duboko i voditi ka povlačenju i autizmu, naročito mlađeg deteta, ili će voditi do poricanja gubitka.

23. Adhevzivna identifikacijaE.Bick - uvodi pojam „adhevzivna identifikacija“, čije je obeležje zavisnost i potreba za slepljivanje sa površinom objekta, kako bi se izgradila „psihička koža“.“A-I“ predhodi pounutrenju delimičnog objekta.

24. Prvobitno i naknadno potiskivanjePotiskivanje ima zadatak da spreči prodiranje mentalnih predstava o nagonima i nagonske težnje u svesnost kao i spoljna zbivanja koja bi mogla da pokrenu strepnju.Razlikujemo prvobitno i naknadno potiskivanjePrvobitno - koristi se rano da se obuzdaju prvobitni izdanci ono i pre njihovog dopiranja do praga svesnosti.Naknadno - koristi se sa ciljem potiskivanja sadržaja koji su već tokom razvoja bili u ravni svesnog i protiv uticaja i delovanja nad-ja.Frojd:Potiskivanje se koristi na psihičke predstavnike pulzije i ovi psihički predstavnici, neprihvatljivi za ja, postaju odbačeni izvan svesnog u prvi stadijum koji je frojd nazvao primarno potiskivanje.ubuduće, svaki događaj, svako uzbuđenje, svaka ideja koja je bila povezana sa potisnutim psihičkim predstavnikom putem asocijacije, trpeće sa svoje strane potiskivanje, isto tako i nizovi ideja, koji proizilaze i koji su povezani sa predstavnikom pulzija, biće odbačeni iz svesnog procesom koji je nazvan sekundarno potiskivanje.25. Izdvajanje – izolacijaPomoću njega osećanja se izdvajaju od misli i udaljuju od ostalih sadržaja duševnog života.

5

Page 6: NEVENKA TADIC PDM - skripta

Ovako otcepljena i izdvojena osećanja su potisnuta i dete - omladinac ne nalazi nikakvu vezu između ova 2 sadržaja.Dete iznosi svoje misli a da ih ne dovodi u vezu sa osećanjima koja su im nekada ranije pripadala.Tako se izbegavaju strepnja i osećanje krivice i sprečavaju npr.udarac, polni napad...U normalnom duševnom životu osećanje, misao i radnja su u skladu i povezani.Kad deluje meh. Izdvajanja misao postoji bez osećanja, a dete nije motivisano da učini, ostvari svoju zamisao.Svoju misao o mržnji, zloslutnim željama pa i ubijanju nekog saopštava hladno. Ovaj mehanizam snažno deluje u prisilnoj neurozi, fobiju, psihoza, disocijativni oblik histerije, psihopatija, autizam

26. Koja je svrha intelektualizacije kod omladinacaIntelektualizacija je bliska odvajanju. Zahvaljujući njoj omladinac, a i starije dete, uspeva da osećajne sukobe podvrgne pod sekundarne procese mišljenja i govora i da ih na taj način obradi i ublaži strepnju koja bi proizašla iz sukoba.Ako su meh. odbrane (kao što su potiskivanje, zdvajanje, intelektualizacija) i suviše jaki, afekt gubi svoju f-ju prilagođavanja i ja ostaje ugroženo.

27. Najteži poremećaji roditelja u odnosu na deteRoditelji sa grubim karakternim promenama, naročito paranoidnim, depresivni i plašljivi, kao i roditelj

prividno očuvanog spoljnog izgleda koji uvlače decu u izmenjeno uzajamna delovanja i uticaje imaju jači patogeni uticaj nego drugi tipovi duševno bolesnih roditelja.

Opasnost je utoliko veća ukoliko su oba roditelja bolesna.Ispitivanja su pokazala da su deca roditelja sa hroničnim telesnim oboljenjima sklona samoubistvu,

kao i deca alkoholičara.

28. Čime se objašnjavaju povišeni izgledi za javljanje duševnih smetnji kod dece roditelja duševnih i telesnih bolesnikaUzajamnim uticajima, pounutrenjima i poistovećivanjem sa izmenjenim ličnostima roditelja i delovanjem drugih mehanizama.Osećajnim ositomašenjem i povišenom agresivnošću i bojažljivošću bolesnih roditelja, naročito depresivnih i šizofrenih.Prisustvo duševnog bolesnika u porodičnoj grupi povećava napetost među članovima i sukobe.Odnosi između duševnog bolesnika i njegove porodične grupe , s jedne strane, i društvene sredine s druge strane.Prisustvo hroničnog telesnog bolesnika u porodičnoj grupi menja porodične odnose, povećava strepnju od smrti, menja društvenoi ekonomsko stanje u grupi.Veći broj razvoda, napuštanja porodice, prestupništva i česta bolnička lečenja.Uticaj nasleđa se pominje kao nedovoljno utvrđen uzrok.

29. Usvojenje – kad saopšitiSamo otkrivanje istine nema značaja kao izdvojen čin i treba ga razmatrati u sklopu sveukupnih

odnosa dete-roditelj.Oko 4 godine života dete počinje da se zanima i postavlja pitanja o pravljenju i rađanju beba, pa i o

svom rođenju. To je prilika da mu se otvoreno i bez dramatizacije saopšti da su ga rodili drugi roditelji koji nisu mogli da se brinu o njemu, te da su usvojitelji roditelji koji su to preuzeli.

30.. Razlozi neuspeha u školiNiska intelektualna sposobnost,

6

Page 7: NEVENKA TADIC PDM - skripta

Telesni nedostaci i bolesti,Povišena zamorljivost,Nedovoljna individualizacija nastave za učenike sa osećajnim smetnjama i posebnim teškoćama, naročito u prvim razredima,Prolazni neuspeh, kao posledica prolaznih osećajnih teškoća koje dete ima u grupi drugova, odnosima sa nastavnikom, u porodici, mladići i devojke sa ljubavnim partnerom,Neurotični odgovori, stanja neuroze, mogu da zakoče velike količine energije u rešavanju unutrašnjih sukoba, da umanjuju ulaganja u intelektualne delatnosti, smanjuju pobude i volju za učenje i da dovedu do slabog uspeha.

31. Uzroci intelektualne inhibicijeNeurotični sukobPočetni psihotični procesi

1) Intelektualne inhibicije neurotičnog porekla su aktivne. Kočenje intelektualnih delatnosti je funkcionalno a cilj mu je da se izbegne neurotična strepnja.Kod histeričnih se ispoljava kao intelektualna slabost, smetnje pamćenja i nedostatak sećanja, odnosno kao smetnja učenja i smetnja zadržavanja naučenog.

2) Intelektualna inhibicija psihotičnog porekla je – složenija i pasivna, pored navedenih simptoma se javljaju i smetnje razumevanja, koncentracije pažnje i siromaštva intelektualnih stvaranja. Psihotični učenik nema uvid u svoju neuspešnost i sklon je da poriče.

32. Razvojna odstupanja, prognoza i poremećajiOdstupanja osećajnog, saznajnog i društvenog razvoja koja su pretežno ili isključivo konstitucionog ili urođenog porekla.

Prognoza: Iako su izazvana pretežno urođenim činiocima, to ne znači da su konačna i da nisu moguća preinačavanja i otklanjanja bilo spontano ili u stručnu pomoć.

Poremečaji:Odstupanja od opštih obrazaca sazrevanja,Odstupanja u posebnim oblastima razvoja:

Razvoj pokretnosti,Opažanje sadašnjih draži,Razvoj govora,Saznajni razvoj,Usporeni ili preuranjeno razvijeni obrasci društvenog ponašanja,Prerano razvijena ili dugo odložena zanimanja za polnost,Nepostojanost ili usporen ili ubrzan razvoj sposobnosti za nadziranje osećanja strepnje, depresije, radosti, besa i ljutnje.

33. Navedite prirodu razvojnih odstupanja. Zašto ona ne moraju ukazivati na patologijuPosledica su nedovoljno poznatih i neispitanih unutrašnjih činilaca, ali olakšana ili podstaknuta spoljnim činiocima.Usporen ili ubrzan razvoj neke sposobnosti može biti posledica uzajamnog delovanja snažnih konstitucionalnih sklonosti, neprometnosti čula i velike osetljivosti na delovanje spoljnih uticaja koji urođenu sklonost mogu ili podstaći ili ometati i otkloniti.Na sličan način mogu da deluju prerano rođenje, porođajne povrede i komplikacije, ometenost čula, telesna oboljenja, koji pojačavaju neku urođenu sposobnost ili sklonost.34. Sisanje prsta- Normalno na uzrastu do nekoliko meseci,

7

Page 8: NEVENKA TADIC PDM - skripta

- Do puberteta i kasnije: 1. Usporeni razvoj telesne šeme; 2. Nazadovanje i fiksacija; 3.Usporen razvoj ja; 4. Produženje autoerotske faze.

35. Dve funkcije uzimanja hraneZadovoljstva koja proizilaze iz zadovoljenja biološkihpotreba za hranom

Zadovoljstva koja proizilaze iz dodira sa dojkom, odnosno objektom preko sluzokože usne šupljine i kože oko usta, zadovoljstvo od uzimanja spoljnih objekata, igre usnicama, mljackanja, gutanja...

Preko usta dete koje u prvim danima i nedeljama je nedovoljno pokretno uspostavlja prve odnose sa spoljnim svetom i na taj način ga ispituje.

36. Uzroci anoreksije, razlika primarna i sekundarna Uzroci:

OrganskiPsihički

PsihosomatskiKulturni činioci.

Jedan od ključnih uzroka je u izmenjenim odnosima prema sopstvenom telu

Primarna-rana-konstitucionalna:Dugotrajnija je i uporna, uzroci :rane psihosomatske razgradnjemanjkavost instinkta samoodržanja

Ukoliko nije reč o ranoj anoreksiji uzroke neuzimanja hrane treba tražiti u izmenjenom odnosu dete-majka i dete-okolina odnosno sekundarnoj anoreksiji.Ako je dojka kao parcijalni objekat ili majka kao ceo objekat doživljena kao *loša* zbog njenih krutih stavova, povišene agresivnosti, preterane popustljivosti, plašljivosti, depresivnosti dete će aktivno ili pasivno odbijati da uzima hranu koju mu ona daje. Odbijanje hrane uznemirava već bojažljivu i preterano agresivnu majku, koja se lako osujećuje i koja odgovara novim povećanjem agresivnosti, a preterano popustljiva majka još je popustljivija i traži načine da umilostivi dete da jede.

37. Ličnost majke anoreksičnih devojčicapovišena bojažljivostagresivna (lako se osujećuje)odgovara novim povećanjem agresivnostipreterano popustljiva majka biva još popustljivija i traži načine da umilostive dete da jedemajka koja ne prihvata materinstvoako je i sama anoreksična ili detinjastaograničava dete u sticanju nezavisnosti i kretanjuzbog svoje prisilnosti nameće detetu krute obrasce obučavanja mokrenja i defekacije.

38. K-ke ja i nad-ja mladih sa mentalnom anoreksijomKarakterisrike ja:

Nedovoljno ustrojenoSlabašnoOd unutrašnjih sukoba se brani cepanjem, izdvajanjem, nazadovanjem i projekcijama u sopstveno telo

Karakteristike nad-ja: (ideal-Ja)Preuveličano

8

Page 9: NEVENKA TADIC PDM - skripta

Ispunjeno osećanjem veličine i moćiPovišena težnja ka samoljubivosti, samodopadljivosti, samodovoljnostiIskrivljena shvatanja o sebi podstiču neprekidnu potrebu za proveravanjem sebe i od strane drugih i postaju izvor neprekidnih osujećivanja, sukoba i kidanja odnosa sa drugimaNaglašeno osećanje moći stiče se preteranom pokretljivošću, obuzdavanjem želje za hranom i erotskih želja, intelektualnom uspešnošću.

39. Terapija mentalne anoreksijeMentalna evaluacija (hospitalizacija)

Fizički pregkedTestovi radi isključivanja drugih uzrokaNametnuto, kontrolisano povećanje težine 150-250 gr dnevno

nametnuto, kontrolisano unošenje hranekontrola stoliceograničen pristup leksativimaograničena rizična aktivnost

Bihejvioralne tehnike:Psihodinamička psohoterapija – individualna (uvid u stavove i emocije) i grupna (5-8 članova zajedno sa bulimijom)Kognitivno-bihejvioralna tehnika (nagrada i kazna za povećanje telesne težine, promena ponašanja)Porodična terapija – porodične terapije

Lekovi: obično lekovi za prateće dijagnoze npr. depresiju, neuroleptici, antidepresivi a u težim slučajevima nastavlja se sa bihejvioralnom terapijom.Prognoza:

- Nešto bolja kod devojčica, kod dečaka teža (zbog dubine nazadovanja, smetnje identiteta i povezanosti sa homoseksualnošću)

- U slučaju kaheksije –prognoza neizvesna- 1/3 se izleći- 1/3 s vremena na vreme su anoreksični- 1/3 poremećaj se razvija u neurozu (depresivnu, histeričnu, prisilnu, hipohondričnu), karakteropatiju

ili psihozu.

40. Majke koprofagaPrema Špicovim ispitivanjima one su nesvesno neprijateljske prema detetu i depresivne su.

41. Ličnost gojaznog deta po BrušuČešća kod devojčica i praćena ubrzanim polnim razvojem, za dečake gojaznost je mnogo veća smetnja naročito ako je masno tkivo raspoređeno na sedalnom delu,manja pokretnost,manja prilagodljivost,slabija prihvaćenost u grupi,na projektivnim testovima kod gojazne dece primetna je manjkavost telesne šeme,zbrka u polnom poistovećivanju Brušteškoće da opaža i razlikuje draži iz vlastitog tela, što je sastavni deo mentalnog ustrojstva, teško prepoznaje draži gladi i sitosti.

42. Prognoza gojaznostiTri oblika gojaznosti:

9

Page 10: NEVENKA TADIC PDM - skripta

Gojaznost bez osećajnih smetnji.Gojaznost kao odgovor na potrese i sukobe odvajanja, razvod roditelja, operativni zahvati postepeno se povlači nakon otklanjanja sukoba.Trajna gojaznost koja se javlja u okviru dubljih osećajnih poremećaja (neuroza, psihoza, karaktera) ili gojaznost koja je prvobitnog porekla, a osećajne smetnje su naknadnog. Može da bude način odbrane od psihotične razgradnje.

Terapija:DijetaNagrada / kaznaUvid u stavove i emocijePsihosocijalne promeneGrupna thPorodična thEdukacija

43. Dinamika gojaznostiUzroci su mnogostruki i često udruženij:

Konstitucionalni (urođena proždrljivost, sklonost gojaznosti) dete je gojazno bez obzira na količinu unešene hrane.HormonskiPsihičkiDruštveni

Konstitucionalni činioci se često prepliću sa psihičkim - primećeno je da su deca gojaznih roditelja češće gojazna pa se postavlja pitanje da li je reč o konstituciji ili oponašanju, poistovećivanju...

Pretpostavlja se da upravljanje apetitom, koje je jednom bilo poremećeno, teško može da se dovede u red, čak i onda kada je spoljna draž (npr. – uporni zahtevi majke na uzimanju hrane) prestala.

44. BruksomanijaObredi uspavljivanja (2-3 god.), zatim postepeno izčezavaju.

Obredi su posebne radnje i postupci sa svojim ili tuđim telom i predmetima za vreme uspavljivanja kao što je sisanje palca, usana, jezika, trljanja delova tela, uvrtanja pramena kose, držanje majke za ruku dok se ne uspava. Neka deca traže da im majka peva, čita, priča priče... Ovi delovi tela i predmeti imaju f-ju prelaznog objekta. Ukoliko se zadrže duže ove potrebe predstavljaju znak sporog i mogućeg nesklada razvoja ja u celini.Škrgutanje zubima (bruksomanija), govorenje u spavanju (somnilogija) i smejanje tokom spavanja, pokreti kao što su ritmije glavom i telom koji se često javljaju zajedno i pre 3. god. predstavljaju načine rasterećenja od povišene napetosti i strepnje. Ove radnje obično ne remete ritam spavanja i ako nisu praćene promenama u EEG-u i dr. simptomimane mogu se smatrati patološkim.

45. U kojoj fazi spavanja se javljaju noćni strahoviOko 70-80% noćnih strahova nailazi u prvoj fazi noćnog spavanja, u prvoj fazi rem spavanja.

“Sporo” NREM spavanje traje 70 – 120 min.“Brzo” REM spavanje traje 10 – 15 min.

Kako noćno spavanje odmiče faze “sporog” spavanja postaju kraće a REM faze duže. Za vreme “sporog” spavanja češće se javljaju umokravanja, mesečarenja, noćni strah...

46. Dve funkcije u napadu noćnog straha (Pavor Nocturnum)1.Da se zadovolji unutrašnja i delimično zabranjena želja (npr.privuče pažnja neke osobe)

10

Page 11: NEVENKA TADIC PDM - skripta

2.Da se izdrži kazna zbog zabranjene želje (trpnja od straha).

47. Mehanizam odbrane kod noćnog strahaPomeranje,Projekcija u snu,Poistovećenje sa agresorom, mehanizmi koji se viđaju i kod fobija, pa neki autori noćni strah izjednačavaju sa fobijama

48. Razlika noćnog strah – fobijaNoćni strah se razlikuje od fobija po tome što odbrane ja, u prvom redu potiskivanje, uspevaju da

spreče prodiranje nesvesnih sadržaja i sukoba u budnom stanju.

49. Uzroci i psihodinamika smetnji spavanja1. Psihički uzroci: očiglednu ulogu imaju sukobi spoljnjeg i unutrašnjeg porekla koji izazivaju strah. Spoljni sukobi i neprijatna iskustva kod mlađe, ali i kod starije dece, iz prethodnog ili prethodnih dana, npr. grube kazne, napuštanje, osujećenja, neuspeh u školi... mogu biti neposredni uzrok smetnjama spavanja. Neprijatna iskustva i sveže povrede preplavljuju ja uzbuđenjima koje ne može da sadrži i poveže, te ove ometaju njegove funkcije i održavanje spavanja.- Sveža povređujuća i osujećujuća iskustva pokazuju težnju da se ponavljaju u snovima. Kroz ponavljanje povrede u snu psih. aparat uči kako da ovlada strepnjom koju izaziva povreda. Neprijetna snovna iskustva mogu da ožive stare potisnute sukobe.

2. Uzroci i psihodinamika smetnji spavanja mogu biti i mnogo složeniji i dublji i voditi poreklo iz nerešenih sukoba, svojstvenih svakoj fazi razvoja.- Nerešeni unutrašnji sukobi preedipalnog porekla na mlađem uzrastu su: strah od odvajanja, osećanje krivice zbog oralnih, analnih agresivnuh i erotskih želja.- Starija deca: unutrašnji sukobi preedipnog, edipnog ali i novijeg doba: osećanje krivice i stida zbog nagonskih težnji – agresivnih prema roditelju istog pola, bratu, sestri, a libidnih preme roditelju suprotnog pola.

50. Pijaže – vrste snova kod deceSnovi koji ispunjavaju nezadovoljene želje i potrebe. Npr. deca često sanjaju kako primaju poklone, putuju..Snovi kroz koje se ostvaruju neostvarene želje bez vidljive simbolike.Snovi koji svesno predstavljaju nešto drugo.Snovi koji predstavljaju neki neprijatan događaj, ali sa prijatnim ishodom (kao u igri).Sošmarni snovi ispunjeni strahovima iz kojih dete može da izvlači zadovoljstva ili su bolni.Snovi kažnjavanja ili samokažnjavanja.Snovi koji su izraz organskog podražaja.Pijaže i M.Klajn- dečiji san je blizak igri.

51. Da li se razlikuju dečaci i devojčice koji se umokravaju po osobinama ličnostiDevojčice pokazuju:

Više težnji ka nezavisnostiTakmičenju sa dečacimaAmbicioznije su

Dečaci:Pasivniji: Zavisniji - naročito od majkiManje agresivni: Skloni povlačenju i samoobezvređivanju

Dečaci i devojčice koji se umokravaju suČešće plašljivi

11

Page 12: NEVENKA TADIC PDM - skripta

Imaju noćne strahove i košmareVezani su za roditeljeSkloni nazadovanju, sisanju prstaDetinjasto ponašanje

Po Kaneru postoje :Delatna i razdražljivaMrzovoljna i razdražljivaTvrdoglava i osvetoljubiva deca koja se umokravaju

Soule proučava odnos između dubine spavanja i umokravanja. Razlikuje pasivnu decu koja više vole da *zatvore oči* pred onim što se dešava, deca koja se suprotstavljaju i ravnodušna su, deca koja mokre u postelju kako ne bi išla na noćni sud i premeštale se u hladnu prostoriju, nastrana deca koja u umokravanju tražezadovoljstvo.

52. Uzorci enureze i na osnovu čega se postavlja dijagnozaUzroci:

Osećajni činiociPorodica, društvena sredina i umokravanjeDruštvena sredina i mokrenjeDuboko spavanje i umokravanjePotomanija i umokravanjeOstali uzroci

neuravnoteženost neurovegetativnog sistemapovišena napetostmali obim bešike

Postavljanje dijagnoza:Laboratorijsko ispitivanjeNeurološko ispitivanjeMetaboličko ispitivanjeRadiografsko ispitivanjeBakteriološkoUrološkoUltrazvučno...

Prognoza :Najčešće prestaje do puberteta, retko se održava do mladosti još ređe u odraslom dobu. Ako se javlja u odraslom dobu izraz je dubljih i strukturnih sukoba iz datinjstva. Može da bude udružena sa simptomima i sindromima – prisilnog, fobičnog, histeričnog reda, a može da bude i jedini simptom.

53. F-je umokravanjaPsihoanaliza - metapsihološko značenje: mokraća, voda i amnionska tečnost imaju zajednička svojstva i umokriti se, biti vlažan, može simbolički da izražava prvobitnu želju za vraćanjem u matericu i plivanju u tečnosti majke.Način suprostavljanja okoliniUživa u prljanju, biti prljav je vlastiti čin deteta i njegova je voljaTežnja ka pasivnostiTežnja ka kožnim zadovoljstvima koja obezbeđuje toplo i vlažno rubljeDepresivni odgovor - plač kroz bešiku - na gubitak objekta ili odbacivanjeIzraz moći i snage, nadmoći nad okolinom

12

Page 13: NEVENKA TADIC PDM - skripta

Suparnički i osvetnički činZamena za polno samozadovoljavanje...Za dečake je čin muškosti, simbol snage i način poistovećenja sa muškom slikom a kod devojčice mokrenje u stajaćem položaju jenačin poistovećenja‚sa muškom ulogom ili takmičarski stavovi u igri među polovima.

54. Simbolička f-ja enureze kod devojčicaDevojčica se oseća uskraćena, zbog toga što nema muški polni ud i činom mokrenja poriče svoj nedostatak. Za nju umokravanje predstavlja suparnički i osvetnički čin.

U edipovom periodu dečaci doživljavaju strah od kastracije. S jedne strane dečak treba da izađe iz stanja bebe i uđe u svet muškosti, a s druge strane nailazi na zabrane od strane porodice da zadovolji svojegenitalne potrebe. Ova dvojnost odnosa podstiče nazadovanje dečaka na ovom stadijumu koji nesvesno zadovoljava kroz mokrenje. U pubertetu umokravanje može predstavljati zamenu za polno samozadovoljavanje. Dečaci dožive erekciju u toku mokrenja i postoji veza između mokrenja i izbacivanje semena.

55. Najpoznatije tehnike lečenja enurezeBihejvioralne intervencije:

Važbanje da se ostane čistStimulisanje da se ostane suvRaspored korišćenja toaletaUvežbavanje kontrole zadržavanja mokraćeObeležavanje donjeg veša

Porodične intervencije:Porodična terapija (sistematska)

Psihoterapija:Terapija kroz igru

Lekovi: Imipramin

Bihejvioralna terapija:Trening suvog kreveta – Azrim je autor, dušek sa dva sloja, dete se umokri, ustane, promeni posteljinu i nastavlja da spava.Pozitivna praksa – dete se samo presvuče, stavlja se da leži 3 minuta pa se dovodi u WC, pokušava da urinira, pa se ponovo vrati u krevet. Tako 8 do 20 puta.

Urinoalarm tehnika – jedna od uspešnijih, stav roditelja treba biti neutralan.Uslovno preplavljivanje – dete pred spavanje napiti vodom.Kontrola bešike – daje mu se mnogo vode tokom dana, pa se pokušava sa kontrolom bešike.Tehnika buđenja – dete bude roditelji i teraju ga da urinira.

56. Fobija od defeciranjaFobija se ispoljava kao jak strah, praćen fiziološkim promenama. Javlja se onda kada dete ima potrebu

za pražnjenjem i kada oseti prva grčenja čmarnih i drugih mišića ili pokreta creva ili kada roditelji pokušaju ili nagoveštaj stavljanja na sud. Dete je zaplašeno, plačljivo, na sve načine se brani da izbegne ovu radnju. Zadržavanje izmeta može da traje danima, kod neke dece potrebno je pražnjenje veštačkim putem što pojačava strah i nemir. Obeležja ličnosti: povišena plašljivost, povišena zavisnost, nesigurnost, od roditelja, pasivnost, zakočena

i pritajena agresivnost, osećajna nezrelost, smanjena podnošljivost na zahteve okoline, dvojnost osećanja prema ocu.

Uzroci – mnogostruki.

13

Page 14: NEVENKA TADIC PDM - skripta

Uloga roditelja:rano stavljanje na noćni sud,prisila i stega,preterano zaštićavanje deteta i zadržavanje u detinjastom položaju u porodičnoj grupi,neprijateljstvo u odnosima,stroge kazne na propuste,iskrivljena opažanja i fantazmi o samom izmetu i činu.

Organski činioci:oštećenja cns-a,organski megakolon (zadržavanje izmeta nekoliko dana pa se smenjuje nevoljnim prežnjenjem),mala telesna težina i visina,polipi creva,bolesti crevnog trakta.

Prognoza i tok: povoljan odnos između roditelja i deteta, nakon otkrivanja i otklanjanja stava koji izaziva poremećaj, ima pozitivan učinak na tok i prognozu.

57. Averzivna terapija kod enkompreze, kada se koristiKazna – time out:ograničavanje aktivnosti koje dete voli, uklanjanje lutke koju dete voli na neko vreme, teranje na odlazak u WC posle jela...

58. ENKOMPREZA – tehnike lečenjaMedicinske:

1) purgativi (lekovi za čišćenje), klistiri i čepići, u slučaju zatvora,2) promena ishrane (dijete).

Bihevioralne intervencije:1) samokontrolisanje (nadzor,praćenje),2) raspored korišćenja toaleta,3) ohrabrivanje – nagrađivanje za čist veš,4) ohrabrivanje – nagrađivanje za korišćenje toaleta,5) time out,6) igra sa kupatilom,7) biofidbek,8) obeležavanje donjeg veša.

Porodične intervencije: porodična terapija.Psihoterapija:

1) terapija kroz igru,2) kognitivno-bihevioralna.

59. Navedi najmanje 3 teorije o uzrocima i nastanku tikovaTikovi mogu biti izazvani medicinskim povredama (oparacija krajnika), jakim strahom (napad psa), prisustvovanje neugodnim događajima, telesne kazne, oboljenja naročito nosa, pluća, očnih kapaka. Tikovi kašljucanja javljaju se posle oparacije krajnika.

14

Page 15: NEVENKA TADIC PDM - skripta

Teorija o organskom poreklu tikovaKomplikovane trudnoće, porođaji, oboljenja koja su mogla da ugroze CNS (zapaljenje mozga). Organske neurološke promene mogu se videti naročito kod dece kod kojih su tikovi udruženi sa diskinezijama, stereotipijama, grčevima mišića i epilepsijom.

Teorija o psihičkom porekluCorbett - simptomi među tikerima: povišena agresivnost, strepnja, neposlušnost, nastupi besa, poremećaji spavanja, izmenjeni odnosi sa dr., mokrenja, defeciranja, depresija, stidljivost... Poreklo tikova se dovodi u vezu sa porodičnim odnosima - majke : napete, zaplašene, preterano popustljive, nasilne i teže ka savršenstvu.

Psihodinamiško učenje - tik, izraz potisnutog sukoba koji je nastao u porodičnoj sredini. Sukob je začet oko agresivnih težnji i njihovog ispoljavanja.

Teorija o psihomotornom poreklu

Deca “tikeri“ - smanjena mogućnost da nadziru neuromišićni rad i zbog toga ispoljene agresivne težnje nalaze način, naročito kod dečaka, da se ulažu u mišiće.

Ostale teorije - teorije komunikacije - tik je dijalog između deteta i roditelja u kome je dete sprečeno da ispolji svoju agresivnost.Prognoza tikova :- najveći broj tikova traje nekoliko dana, nedelja a ređe se produžavaju i traju mesecima- Tuterova bolest – traje godinama pa i ceo život- najbolju prognozu imaju tikovi koji počinju između 7 i 8 god.

60. Sa kojim poremećajima se javlja tikDiskinezije Prisilne neurozeStereotipije Hiperkinetički sindromGrčevi mišića Usporen razvoj pokretnosti.Epilepsija

61. Uzroci hiperkinetičkog sindromaUzroci mogu biti psihički koji deluju u okviru porodice i škole, organski, neurofiziološki, genetički i dr.Manjkavi objektni odnosi i prvobitna nezadovoljenja teraju dete da neprekidno traži zadovoljenja. Uzroci slabih obj. odnosa mogu ležati u detetu i u roditeljima koji ne zadovoljavaju nagonske potrebe svoga deteta.Prognoza :Verovanje da će hiperkinetički poremećaj isčeznuti sa rastom nije uvek opravdano. Nemir, razdražljivost, naglost i slaba pažnja zadržavaju se dugo iako su manje izraženi.Preterana pokretljivost smanjuje se u mladosti. Hiperkinetičkom sindromu kod nelečene dece i pod nepovoljnim porodičnim i društvenim uslovima naknadno se pridružuju :Karakterne promene, nedruštvena ponašanja, depresivnost i profesionalne teškoće.

62. Kada je indikovana psihodinamska psihoterapija kod hiperkinetičkog sindromaTerapija igrom – psihodinamska psihoterapija:

za mlađu decu:1) rešavaju nisko samopoštovanje,2) za anxiozne depresije,

15

Page 16: NEVENKA TADIC PDM - skripta

3) ljutnju na autoritete,4) frustraciju – agresiju.

za stariju decu:psihoterapija podrške.

Ostali tretmani:najčešće su dijetalne intervencije,neki smatraju da ovaj sindrom nastaje zbog trovanja olovom – živom, ali ređe da ovo trovanje može izazvati trajnije simptome.

63. Najefikasniji lek kod hiperkinetičkog sindroma?R i t a l i n

Psihostimulansi (ritamin,......., cylert.)Triciklični antidepresivi (nokpamin, tofralin)Ostali lekovi (klonidin, tirodazin)

64. Psihoterapija hiperaktivnog poremećajaBihejvioralne intervencije:

U kući / van kuće (ordinacija)PsihoedukacijaVežbe pažnje „posebno vreme“ - vreme koje roditelji kao deo terapije provode sa detetom (15-20 min. dnevno) u međusobnom spontanom druženju i interakcijiObeshrabrujuće (stimulativne) tehnike (nagrade-kazne)Tehnike relaksacije

U školi1) Redukcija stimulansa (situacije koje izazivaju nepažnju, impulsivnost ili hiperaktivnost, npr.

stavljanjem deteta da sedi na neko mirno i tiho mesto)2) Promena mesta sedenja (pomeranje da sedi što više napred u prvi red ili što dalje od dece koja su

previše aktivna ili se potpuno može promeniti raspored sedenja u učionici)3) Povećanje broja zadataka (obaveza) i stimulativnosti zadataka4) Reagovanje u zavisnosti od ponašanja deteta u cilju oblikovanja njegovog ponašanja

Psihoterapija:Kognitivno-bih. intervencije

self talksamoposmatranjerešavanje problema korak po korak od strane detetasterapija kroz igru

Napomena – specifična lečenja se često kombinuju u paket intervencija

65. LatencijaStadijum mirovanja nakon edipne-falusne faze. Počinje nakon 5.–6. godine i traje do ulaska u pubertet.

Uništenje(m) edipovog složaja i oslobađanje energije potrbne za druge delatnosti ja, na primer intelektualne. To je faza u kojoj je libidna energija oslobođena za intelektualne delatnosti, za stvaranje prijateljstva,

16

Page 17: NEVENKA TADIC PDM - skripta

bavljenje umetnošću i drugim višim delatnostima. U ovoj fazi su mehanizmi potiskivanja i poistovećivanja posebno živi. Kako dete ne može da pobedi roditelja istog pola poistovećuje se sa njim.

Posebno mesto zauzimaju reaktivne formacije koje stvaraju snažna protivulaganja (kontrainvestiranja).

66. Psihodinamika i uzroci preteranih masturbacijaPatološka masturbacija ima sledeće odlike:

Učestala,Prisilna obeležja,Isključivi vid polnog zadovoljenja,Zavisnost koja odvlači veliku ili svu količinu libida.

Patološka je jer otežava i onemogućava dalji razvoj polnosti, pa i prilagođavanja.

Preterano samozadovoljavanje u mladosti ima dublje uzroke:

Preterno podsticanje, zavođenje i prerano upućivanje na polnost od strane roditelja i drugih osoba.Slabi odnosi sa roditeljima i drugima.Izraz strepnje i poremećaja obj. odnosa, prvenstveno sa majkom

Izazivaju:Jaka strahovanja i strepnjuOsećanje krivice i stidaHipohondrične tegobeDalje slabljenje društvenih veza

67. Uzroci masturbacijeDete dosta rano otkrije da neki delovi tela obezbeđuju zadovoljstvo, bilo da ih dodiruje drugi ili ono

samo. Tokom prve godine dolazi do stezanja butina, ljuljanja, trljanja polnih delova rukama... koji obezbeđuju podražaj genitalnog aparata kod neke dece.

Dodirivanje polnih organa, u prvo vreme slučajno, a kasnije sa ciljem, ima funkciju ispitivanja tela, da bi nakom otkrivanja zadovoljstva postalo namerna radnja koja obezbeđuje polno zadovoljstvo.

Primarna: tokom prve godine. Sekundarna: u falusnoj fazi (3-5 god.).

Nabreknuće i podizanje polnog uda kod dečaka i dražice kod devojčica koji se mogu javiti oko 1 god. u budnom stanju, spavanja, sisanja praćene su uznemirenošću, plačem, ukrućivanjem tela, promenom boje lica, crvenilom, znojenjem... čini se kao da erekcija izaziva prijatnost i podseća na sladostrašće (orgazam). Posle izvesnog vremena dolazi do smanjenja napetosti. Ova prvobitna samozadovoljavanja traju do 2 god. kada se učestalost masturbacija smanjuje pa se opat povećava sa ulaskom u falusnu fazu. U fazi mirovanja ponovo dolazi do polnih zanimanja i radnji a u pubertetu do novih oživljavanja pretežno na polnim organima.

68. Poremećaji psihosex. identiteta u detinjstvu i adolescencijiPojačana polnostSmanjena polnostPromene u izboru objekta zadovoljenja - polno samozadovoljavanje, homoseksualnost, fetišizam, zoofilija.Promene u načinu nagonskog zadovoljenja - voajerizam, ekshibicionizam, sado - mazohizam, tranveztitizam, transeksualizam.Ostali poremećaji - rani pubertet, polna nazrelost, urođeni nedostaci...

1 - 4 → uzroci pretežno psihički i društveno-kulturni

5 → neuro-hormonski

17

Page 18: NEVENKA TADIC PDM - skripta

69. Psihodinamika i uzroci pojačane polnostiUzroci:

Dublji sukobi u odnosima sa porodicom StrepnjaOsećajna glad SuprostavljanjeDepresivnost Manjkavosti u razvoju ja i nad-ja sistemaKompulzivnost Smanjivanje zabrana od roditelja i društva

Preterano erotizovana ponašanja viđaju se rano kod izvesnog br. mladih u svim slojevima društvadečaci i devojčice rano stupaju u polne odnosečesto menjaju partneredevojčice ostaju rano trudne i postaju majkesvesna-nesvesna erotska ponašanja roditelja-promiskuitet, prostitucija i podvođenje, podstiču oponašanja i poistovećivanja i razvoj polnih želja i težnji, pa i ispoljavanja.Rano prisiljena da prisustvuju polnim odnosima roditelja, dugo spavaju sa njima ili zavođenje od starijih članova porodice. Polna zanimanja i ponašanja su pojačana.

70. Fetišizam kod dece i odraslihZamenjivanje majčinog tela, posebno grudi nekim predmetom koji dete prisvaja. Zamenu nalazi kako u svom telu, tako i u delovima tela drugih, ali i u delovima odeće.Kod odraslih fetiš ima značajnu, čak ključnu ulogu i predstavlja cilj polnog maštanja.U razvoju fetišističkih sklonosti i u izboru fetiša značajnu ulogu igra majka. Ona se nesvesno opire odvajanju od deteta i odbija da zadovoljava njegove telesne potrebe u pojedinim fazama razvoja. Ovaj odnos može da bude uspostavljen kroz fetiš koji predstavlja ili deo tela majke, ili njeno telo u celini.Fetiš ime simboličko, ali i stvarno značenje i obezbeđuje detetu gledanje ili dodirivanje i mirisanje. Dete se na taj način oslobađa rušilačkih, sadističkih poriva koje ulaže u ovaj fetiš – objekat.

71. Ispoljavanje agresivnosti na oralnom, analnom, falusnom stadijumu i kasnijeOralni stadijum: Agresivna i libidna energija prazni se preko usta (griženje, gutanje, proždiranje, ujedanje, pljuvanje), a na višoj ravni razvija se kroz govor.

Analni stadijum: Prljanje, defekacija, ispuštanje vetrova. Tvrdoglavost, svojevoljnost, mučenje, zločestost.

Falusni stadijum: Agresija se ispoljava na manje neposredan i telesni način, a više kroz osećanja neprijateljstva kao što su mržnja, zavist, ljubomora.

Kasnije: Simbolično kroz igru, svakodnevne delatnosti i odnose, sport, stvaralaštvo, rad i nauku.

72. Samopovređivanje - šta predstavlja i ulogaAutoagresivna ponašanja :Pojava koja se tokom razvoja viđa kod svakog deteta grize se, grebe, bocka, češa, čupa kosu...U prvim godinama može da predstavlja :refleksni odgovor na draži, način rasterećenja i oslobađanja napetosti i strepnje, način ispitivanja svog tela, stvaranje telesne šeme i telesnog, a zatim psihičkog ja, način prilagođavanja okolini.Oko 4.godine i kasnije polagano isčezava.Starija deca i omladinci - bezazleno, ali i ozbiljno - gašenje opušaka na rukama, povređivanje očnih jabučica, rezanje vena...Obavlja se stalno ili u nastupima. Omladinac se povređuje mirno i očekuje odgovore okoline - koja je obično iznenadena i zaplašena. Nekada je iznenađujuća neosetljivost na bol, naročito kod umno zaostalih i psihotičnih a najčešće šizofrenih osoba.Ponovno javljanje na kasnijem uzrastu znak je nazadovanja koje je izazvano nekim spoljašnjim ali i unutrašnim činiocima. Ako se autoagresivne radnje zadržavaju dugo znak su vezivanja i učvršćenja u određenoj fazi razvoja i zastoja afektivnog sazrevanja. Tada su vidljive promene u razvoju Ja, poremećaj telesne šeme, nedovoljna razlučenost između Ja i ne-Ja, nedovoljna svesnost o sebi.

18

Page 19: NEVENKA TADIC PDM - skripta

73. Karakteristike osoba koje su pokušale smoubistvoDeca i omladina koja pokušavaju samoubistvo ne ispoljavaju posebna psihopatološka obeležja na

osnovu kojih bi se mogao dijagnostifikovati samoubilački poremećaj kao zasebna klinička celina.

Sledeća obeležja ličnosti ove podgrupe mladih mogla bi biti povezana sa samoubilačkog ponašanja:smanjena sposobnost za govorno saopštavanje unutršnjih sadržaja i misli,sklonost ka prelasku na radnju,nemogućnost mentalne obrade telesne napetosti,preterano korišćenje mehanizma intelektualizacije i racionalizacije koje ne uspevaju da spreče agresivne težnje i ublaže mrzovoljnost, samoljubivost i strepnju,žalost i neprijateljstvo prema sebi i prema svetu.

74. Koje poremećaje svrstavamo u poremećaje navikaPatološko kockanje (kod odraslih),Patološko podmetanje požara (piromanija),Patološka krađa (kleptomanija),Patološko čupanje kose (trihotilomanija).

75. Razlike između neuroza i poremećaja navikaPoremećaji navika spadaju u specifične por. ponašanja koji se karakterišu ponavljanim postupcima koji nemaju jasnu racionalnu motivaciju. Pacijent saopštava da je patološko ponašanje povezano sa impulsima koje on ne može da kontroliše.U neurozama konflikti su nesvesni, a o njihovom postojanju zaključujemo na osnovu ponašanja pacijenta.

76. Anksioznost po različitim stavovimaStrepnja ili teskoba ili strah (bojažljiv, plašljiv)O. Rank → porođajno poreklo. Smatra da je rođenje prva opasna ,doživljena okolnost koja predstavlja povredu i jak stres za dete kako u fiziološkoj, tako i psihičkoj ravni. Iskustvo rođenja je izvor doživljavanja strepnje u toku kasnijeg života.

S. Frojd → posledica neposrednog pretvaranja libida zbog osujećenja i potiskivanja polnog nagona, da bi kasnije, sasvim izmenio svoje shvatanje i izjavio da strepnju izaziva unutrašnja napetost. Libidnog i agresivnog porekla i da zapravo strepnja pokreće potiskivanje.

M. Kajn → strepnju prvobitno dovodi u vezu sa sukobima između nagona života i smrti koji se odvijaju u okviru ono, a zatim sa paranoidno-šizoidnom pozicijom i projekcijom agresivnih težnji u bliske objekte, što je izvor strepnje progonjenja i sa depresivnom pozicijom koja je izvor depresivne strepnje. Strepnja je za nju unutrašnjeg porekla.

M. Maler → strepnja je izazvana prekidom simbiotskog odnosa između detete i majke i doživljavanjem nastajanja, a zatim procesima odvajanja i individuacije koji se odvijaju u 3 faze.

H. S. Salivan → smatra da bojažljiva i nesigurna majka neposredno, putem empatije, pogleda, glasa prenosi strepnju na svoje dete.

77. Teorije nastanka anksioznostiTeorija učenja

19

Page 20: NEVENKA TADIC PDM - skripta

Dete koristi strepnju da bi se povuklo iz normalnih društvenih međuodnosa i međudelovanja. Zastrašujuća iskustva mogu izazvati strepnju. Kako su iskustva strepnje neprijatna pojačavaju se izbegavajući odgovori koji treba da smanjuju strepnju. Ovi neprilagođeni odgovori menjaju ponašanje i društvena međudejstva.

Egzistencijalistička školaStrepnja nije naučeno iskustvo, već je prirodno stanje organizma koje se javlja od početka života kao deo ljudskog postojanja i podstrekač za rešavanja osnovnih pitanja čoveka...Istinsko ljudsko postojanje se suočava i podnosi egzistencijalnu strepnju.

Kulturna psihijatrijaRazličito se objašnjava. Govori se o osnovnoj strepnji koju dete iskusi zbog osećajne nesigurnosti i neprijateljstva koje dolazi iz spoljne sredine, ravnodusnošt, nemoralnih ponašanja odraslih, prevelike ili premale odgovornosti... Hornaj

78. Klinički oblici ispoljavanja strepnjePovremeni napadi strepnje izazvani povredamaJaka strepnja se javlja u napadima nakon jedne ili više ponovljenih povreda (hiruški zahvat, u stanju visoke temperature, psihički stres). Ovi odgovori traju najčešće za vreme povrede i nastaju kada se dete oseća bespomoćnim i nezaštićenim. Praćeni su razdražljivošću nesanicom, zastrašujućim snovima i depresijom. Nastupi strepnje isčezavaju otklanjanjem osnovnog uzroka odnosno povrede. I pre nastupa strepnje kod ove dece postojala je povišena spremnost da se odgovori na stres, povišena osećajna osetljivost i zabrinutost.

Napadi strepnje bez vidljivog povodaJednom ili više puta tokom dana dete-omladinac postaje uznemiren, veoma uplašen, ukočenog pogleda, širokih zenica, smetnji disanja, srca, probavnog i mokraćnog sklopa i sa znojenjem. Sve počinje naglo i traje par minuta do pola sata. Najčešće bez vidljivog povoda. Neposredni povodi napada strepnje su smrt u porodici ili bliskoj okolini, neprijatna iskustva sa osobama, životinjama... Ličnost ove dece : povišena nesigurnost, zavisnost, plašljivost, osećanje krivica, neprijateljstvo prema drugima, a porodična atmosfera je kruta i agresivna. Simptomi postepeno isčezavaju ili se organizuju u prave neuroze. Kod mlađe dece ispoljava se vikanjem, besom, pokretima tela...

Hronična strepnjaNeka deca su trajno u stanju ošenivanja opasnosti, zabrinutosti, zaplašeni su, lako se trzaju, napetu su, ponašaju se kao da ih opasnost vreba sa svih strana. Čest je tremor prstiju, grickanju noktiju i usana... Hronična strepnja osiromašuje međuljudske odnose osobe, smanjuje težnje, remeti saznajne procese... Iz hronične strepnje mogu se razvijati, u početku nesistematizovane a kasnije i sistematizovane fobije.

Strepnja od odvajanjaKada majka napusti svoje dete uzrasta 6-8 mesecimakar i na kraće vreme u fazi simbiotskog odnosa i kad je u prisustvu nepoznate osobe dete ispoljava *strepnju od odvajanja*. Dete doživljava odsustvo majke kao opasnost jer se bez nje oseća bespomoćnim i preplavljeno je strepnjom. Strepnja je posledica odvajanja i nezadovoljenja njegovih unutrašnjih potreba na koje može da odgovori samo majka kao stalni objekt.

Tri načina ispoljavanja strepnje :- Pre 3 meseca reč je o stanju fiziološke napetosti i trpnje. Teškoća se javlja kao posledica poremećene unutrašnje energetske ravnoteže u telu.- Posle 3 meseca zahvaljujući tragovima pamćenja doživljenih neprijatnih iskustava pojavljuje se odgovor straha koji je vezan za objekt iz okoline.- Između 6-8 meseca kada dete opaža majku kao celovit i stalni objekat i kada počinje da razlikuje strane od bliskih osoba pojavljuje se strepnja kao složena psihička pojava.

20

Page 21: NEVENKA TADIC PDM - skripta

Bolbi – strepnja je prvi glavni odgovor deteta na odvajanje od majke za koju je ono nagonski vezano. Strepnja 8. meseca je normalna pojava u ranom razvoju i dokaz uspostavaljanja objektnog odnosa sa majkom. Ako se ne ispoljava znači da majka nije doživljena i opažena kao značajna i celovita osoba.Ako je strepnja od odvajanja previše jaka i dugotrajna i bez odgovarajuće zamene ostaje kao obrazac neprijatnog iskustva koji se može ponavljati kasnije u životu u sličnim i istim okolnostima odvajanja.

79. Na kom uzrastu počinjemo da govorimo o dečijim fobijama umesto o strahuNajveći broj strahova kao i strepnja javljaju se normalno u raznim fazama života i polagano

isčezavaju sa sazrevanjem, stvaranjem primerenijih odbrana ja, i sticanjem stvarnih iskustva koja ispoljavaju maštu i nepotpunu procenu stvarnosti.

U tom slučaju se može govoriti o normalnim fobičnim odgovorima ili stanjima. Tek kada se ovi fobični odgovori uobličavaju i organizuju, kad celokupna ličnost deteta i omladinca poprima neurotičnu organizaciju sa nesvesnim sukobom u pozadini, nazadovanjem, teškoćama u funkcionisanju i ostalim simptomima, može se govoriti o organizovanim fobijama. Ovo se ne viđaju u ranim mesecima i godinama života, ali se njihovi prapočeci mogu nazirati već u edipnoj i fazi mirovanja.

80. Po Bakeru, na kom uzrastu se javljaju fobije: 11-14 godina

81. Koji su mehanizmi odbrane kod fobijaFobije:

premeštanje, simbolizacija,sažimanje, projekcija,

- Opasnost biva premeštena u spoljni svet objekata daleko od unutrašnje stvarnosti.- Premeštanjem se prenosi energije sa jednog objekta na drugi, u slučaju fobije na spoljašnji koji na

simboličan način predstavlja zastrašujući objekt.- Projekcijom se opasnost, koja je sada spoljašnja može izbeći, što fobična osoba i čini. Projekcijom se

izbacuje iz sebe *unutrašnja opasnost* instinktivnog porekla i ubacuje u spoljašnji svet.Fobije se lakše javljaju kod onih kod kojih potiskivanje nije dovoljno jako i kod kojih su se mehanizmi

projekcije i premeštanja zadržali duže

82. Obeležja ličnosti sa školskom fobijomPovišena nesigurnostPlašljivostPovišena povredljivostSklonost ka nazadovanjima i projekcijamaSklonost ka depresivnim i fobičnim odgovorimaZavisnostSamoljubivostUzroci : smetnje u organizaciji ličnosti, izmenjen odnos dete-roditelj, društveni i kulturni činioci, visoki zahtevi škole, kruta školska sredina i ličnost vaspitača.

83. Na kom uzrastu možemo govoriti o prisilnim neurozama

21

Page 22: NEVENKA TADIC PDM - skripta

Rano, posebno oko 3.god. života se javljaju brojni obredi uspavljivanja, kao što su klaćenje, vrćenje čuperka kose ili uha prstima, sisanje jezika i usnica, uzimanje predmeta-igračke u postelju. U pozadini ovih obreda leži strah od odvajanja i smrti.

U fazi mirovanja, kada se pojavljuju i nameću mnoga ograničenja i zahtevi porodice, šireg društva i škole, javljaju se i organizuju se novi obredi, prisile i kompulzije: pokreti rukama, skakutanje, brojanje, pevanje, ponavljanje reči...

U pubertetu i mladosti ispoljavanja su sve složenija i sličnija tome što se viđa u odraslom dobu. Ova deca u ranom detinjstvu su teška i nemirna, imaju povišenu strepnju, fobičnost i histeričnost. Pojava opsesivnih neuroza u pubertetu roditelji ocenjuju kao poboljšanje ponašanja.

84. Fobične opsesijeU njima se javljaju protumere čija je f-ja zaštita od fobičnih doživljavanja.Nije uvek jasna granica izmeđ fobičog i prisilnog sindroma, ali je jasna u starijem uzrastu.Kad je fobičost veoma jaka i kad zataje odbrane kao što su premeštanje, potiskivanje i izdvajanje, a strepnja i fobija preplavljuju ja, koriste se mnoge radnje koje poprimaju kompulzivna svojstva sa ciljem odbrane od nedozvoljenih težnji i nadziranja fobogenih osoba, predmeta i okolnosti.Protumere - paljenje svetla kod fobije od mraka, mirisanje hrane kod fobije od hrane ili prisilno bavljenje sportom, mogu da otkriju poreklo potisnutog sukoba.Ove protumere nikada ne uspevaju potpuno i trajno da odstrane napetost i strahovanja.

85. Simptomi impulzivne opsesijeGrupa impulzivne opsesije ispoljene kao :Prisilni strah od besmislenih radnji - izgovaranja skaradnih reči, ispuštanja vetrova, mokrenja na javnim mestima...-Strah od izvršene radnjeStrah od zadovoljenja pulzije i željeJavljaju se različiti por. Ponašanja - kleptomanija, bekstvo, napadi besa, ekshibicionizam...- i ovi poremećaji mogu da budu opsesivno-kompulsivno impulsivnog porekla.

86. Mahanizmi odbrane kod opsesivno-kompulzivnih neurozaOpsesivno-kompulzivne neuroze:

potiskivanje, intelektualizacija,izdvajanje, reaktivna formacija,nazadovanje, acionalizacija

87. Kakva je prognoza opsesivno-kompulsivnih neurozaRazvoj prvih o.k. ispoljavanja u detinjstvu je neizvestan. Pod povoljnim okolnostima mnoga od njih

su prolazna, učestvuju u stvaranju i ulaze u sastav ja, koje poprima manje-više prisilna obeležja, ali u okvirima normalnosti.

Pod nepovoljnim okolnostima stvara se prisilni k-kter sa naglašeno prisilnim obeležjima do krajnje neobičnih razmera.

Ređe se javlja i razvija prisilno stanje koje se sa povremenim promenama u načinu ispoljavanja, poboljšanjima i pogoršanjima može organizovati u pravu prisilnu neurozu već u detinjstvu i zadržati do odraslog doba.

Kad svi nesvesni meh.odbrane, kao što su izdvajanje, potiskivanje, premeštanje, intelektualizacija, racionalizacija i reakt.formacija otkažu, postoji opasnost da nazadovanje dosegne psihotičnu ravan.

88. Konverzivni – senzitivni poremećaj

22

Page 23: NEVENKA TADIC PDM - skripta

Gubitak osetljivostiBolovi i preterana osetljivost pojedinih delova kože i sluzokože,bez odgovarajućeg organskog nalaza, javljaju se češće kod omladine.Kod dece su veoma retki, jer mlado dete teško doseca i zamišlja da postoje i smetnje osetljivosti. Viđaju se u obliku-gubitka osetljivosti kože ruku.Neki omladinci gase cigarete na rukama, bockaju iglama i pri tom ispoljavaju upadljivu neosetljivost.Mehanizam nastajanja konverzivne neosetljivosti udružuje se i ispreplićce sa mazohističkim i drugim težnjama. Samopovređivanjem i ispoljavanjima.Opisane su neosetljivosti u obliku rukavica ili kratkih čarapa, polovine tela hemianestezije.

89. Koji konverzivni poremećaji potiču iz pregenitalne faze fiksacijePoremećaji glasovnog aparata (afonija) praćeni oduzetošću usana i jezikaSmetnje disanjaŠtucanjePovraćanjeSmetnje gutanja (disfagije)Smetnje spavanjaSmetnje mokrenja (polakiurije)Anoreksija i bulimija mogu biti i konverzivnog porekla

90. Uzroci nastajanja mutizmaDešava se da deca zaćute u :novim i zastrašujućim okolnostimaprisustvu odraslihstanja iznenađenjaza vreme ispita, ali ova kratkotrajna ćutanja ne mogu da se uvrste u tipične konverzije, ali mogu biti konverzijama sličneako traju duže, dete-omladinac na simboličan način izražava da je govoriti opasno, zastrašujuće i da se toga odričečešća je kod dece sa smetnjama artikulacijedete izvlači sekundarnu dobit (npr. ne odgovara u školi, izbegava odnos sa roditeljima koji kažnjavaju).

91. U kojoj fazi psihomotornog poremećaja nastaju konverzivni poremećajiS. Frojd – u falusnoj fazi

92. Izbor konverzivnog organaIzbor organa vrši ja, odnosno njegove odbrane i primarni procesi mišljenja, zavisno od predhodnog

iskustva sa pojedinim delovima tela i energije uloženo u organ.Kod sasvim malog deteta, zbog slabosti ja, izbor nije tačno određen i trajan. Telo u celini postaje

organ izbora i način saopštavanja.

93. Koji su osnovni meh. odbrane prisutni kod nastanka konverzijaPremeštanje SažimanjeProjekcija Simbolizacija

94. U čemu je razlika između konverzivnih i disocijativnih reakcija i stanja

23

Page 24: NEVENKA TADIC PDM - skripta

Disocijativni odgovori: izražavaju napor ja da se izdvoji i udalji od nekih sadržaja koji izazivaju strepnju od koje ja ne može da se brani na neki drugi način. Po ovome su bliski prisilnim odgovorima.

Od konverzivnih odgovora se razlikuju po tome što dete-omladinac ne pokazuje javno svoj sukob preko tela već želi da ga izbegne izdvajajući ga i od okoline i od sebe samog.Ova razlika nije uvek jasna, jer neke osobe koje odgovaraju na ovaj način, traže prusustvo drugih, i javno, u maskiranom obliku, iznose svoj sukob i svoje želje.Disocijativni odgovori : otuđenje (depersonalizacija), amnezija, dnevna sanjarenja, mesečarenja, psihogeni stupor...

95. Somnabulizam - uzrok i prognozaPosledica je sukoba između želja i odbrana ja i naj-ja sistema. Javljaju se tokom spavanja, češći su u

detinjstvu nego u odraslom dobu. Ja mesečara nadzire kretanje i obično ne dolazi do povreda iako su povrede moguće.

Kod dece su sukobi “mlađi” i posledica su izmenjeni objektnih odnosa, nezadovoljenih potreba za ljubavlju, sigurnošću, polnih i agresivnih težnji.

Prognoza:u najvećem broju slučajeva procesi sazrevanja i razvoja imaju prevlast i mnogi sukobi se dešavaju drugim i primernijim mehanizmima, abreagovanjem, sublimacijom, potiskivanjem, reaktivna formacija i drugim načinima pa ne dolazi do organizovanja neuroza.u slučaju slabije integracije ja u određenim nepovoljnim okolnostima omladinac, pa i odrasli odgovore na način sličan onom u detinjstvu, i ako se još ne može govoriti o neurozi.pod nepovoljnim okolnostima, konverzivno-histerični odgovori se ustaljuju, njihov osnovni obrazac i jezgro učvršćuju, pa se tako organizovana preneuroza, ranije-kasnije preobraća u konverzivno-histeričnu neurozu.

96. Ličnost histerične dece i omladinepovišena detinjatost želja da se bude zapažen i da mu se dive.povodljivost osetljiv na osujećanja, gladna osećanja.samoljubivost teško podnosi dugotrajne napore.sebičnost normalne inteligencije.spremnost ka nazadovanju sklonost ka - maštanju, dnevnom sanjarenju, uzbuđivanju,

teatralnosti, laganju, mitomaniji i klevetanju.

97. Koji su kriterijumi razlikovanja konverzivnih od psihosomatskih poremećajaAko poremećaj unutrašnjih organa nema simboličko značenje, nego predstavlja neposredno

rasterećenje, to se ne može shvatiti kao odgovor na strepnju ili kao psihosom.poremećaj.Razlika nije uvek uočljiva. Oba poremećaja nalaze telesni način izražavanja i donose sekundarnu

korist. Konverzija je izraz unutrašnjeg sukoba i bez telesne promene, a psihosom.oboljenja neposredno telesno izražavanje afekata ili potisnute težnje sa telesnim promenama.

98. Roditelji deteta sa konverzijamaRoditelji koji snažno osujećuju biološke, osećajne, saznajne i društvene potrebe svog deteta.Erotizovani histerični roditelji koji erotizuju odnose sa svojim decom u svim fazama razvoja.Mazohistički roditelji.

99. Razlika konverzija – fobija

24

Page 25: NEVENKA TADIC PDM - skripta

Sličnosti – u obema se koriste mehanizmi: 1. Premeštanja, 2. Sažimanja, 3. Simbolizacije, 4. Projekcije

Pacijenti i fobični i histerični projektuju izdanke svojih nesvesnih fantazija u nameri da se oslobode neizdržive napetosti i strepnje.

Razlika – fobično dete-omladinac svoje fantazije projektuje u spoljni svet, objekte i okolnosti, pa opasnost kao da dolazi spolja i kao da je jasna nejednakost između unutrašnje i spoljašnje delatnosti

U konverziji dete-omladinac svoje fantazije projektuje uglavnom u svoje telo, kao da postoji nejednakost između unutrašnje i telesne stvarnosti

100. Konverzivni sindromKonverzivni sindrom je simbolski izraz nesvesnog mentalnog sukoba. Nesvesne i potisnute instiktivne želje prete da dopru do svesti. Konverzivni simptomi predstavljaju odbrane od instiktivnih želja od kojih se ja brani. Sadržaji sukoba su potisnute libidne težnje koje su se razvijale do edipalne faze. Histerični sukob ima poreklo u falusnoj fazi.

101. Terapija konverzije1) Psihoterapija, 6) Sredstva za smirenje,2) Placebo, 7) Nagrađivanje,3) Hipnoza, 8) Relaxacija,4) Sugestija, 9) Bih. modifikacija,5) Povećanje samopouzdanja, 10) Porodična terapija.

102. Intervencije porodice i škole kod konverzivnog poremećajaPorodične intervencije:porodica utiče na izbor organa kroz koji će da se izrazi simptom. Porodica smatra da deca normalno fun-šu u socijalnom, intelektualnom i emocionalnom smislu i opiru se psih. objašnjenjima konverzivnih simptoma, imaju velika očekivanja od dece, veliki nivo kontrole i vezanosti – ispod kojih su osnove osećaja straha da porodica nije baš „normalna“, te da bi na izgled sve izgledalo normalno i pod kontrolom, izbegavaju se jake emocije, a ispoljavaju se kroz telesne probleme.

103. Kognitivna terapija kod depresijaKognitivna restrukturacija:

Ohrabruje decu da situaciju interpretiraju na drugi način. Dete ima drugačiji pogled, pa je situacija manje katastrofalna, manje značajna ili čak pozitivna.Nadolazeća teškoća se sada posmatra kao izazov za uspeh, mogućnost za sticanje iskustva i prethodna iskustva se tumače u pozitivnom svetlu.Ovo je moguće i za mlađu decu jer terapia predlaže novi okvir situacije, a dete treba samo da je prihvati.

104. „Holding“ tehnika i mišljenje kod autistične dece„Holding“: integracija potpomognuta držanjem i posedovanjem → dete stiče osećanje stalnosti i povezanosti zavisno od sigurnosti koju mu obezbeđuju osobe – roditelji. Holding štiti dete od fizioloških opasnosti. Zaštitu pružaju roditelji danju i noću, svojom brigom za telo deteta (preko dodira, temperature, sluha i vida).Poremećaj toka i sadržaja mišljenja – ili se mišljenje neće razviti, a ako se razvije karakteriše ga:

Sporost,Lepljivost,Prekidanje misaonog toka,Raspadanje misli i mišljenja,Čudne misli, zaokupljenost..

105. Autizam gde je očuvana inteligencija

25

Page 26: NEVENKA TADIC PDM - skripta

Kannerov sindrom

106. Govor autistične decePostoji opšte kočenje i retardacija govora ili kognitivnog razvoja. U terapiji dete se nagradi za bilo koji vokalni izgovor, samo kad vokalizacija sledi govor terapeuta. Dete uči fenomene i njihovo kombinovanje u reči, razmenjivanje kad dete nauči da simbolizuje upotrebu reči, rečenice.

107. Autistična deca sa većim IQ-omKaner – IQ autistične dece je najčešće normalan, a niži ravan intelektualnog f-nja pa i niži IQ posledica su poremećenog odnosa sa svetom. 1/3 dece nema oštećen IQ

108. Autizam – tehnike lečenjaPorodične tehnike: obrazovanje roditelja o autizmu, njegovom uticaju na stres roditelja i njihove emocije; savladavanje stresa roditelja.

Bihejvioralne tehnike:

Smanjivanje mešanja (interfering) i opesnog ponašanja:izolacija (time-out)promena različitih drugih ponašanjapozitivna praksaignorisanjetehnike kažnjavanj

Učenje tehnikama spremnosti (prihvatanja)učenje da su odrasli izvor prisileslušanje odraslihučenje da se izvrši naredbaimitacijakorišćenje jezika

Učenje društvenom ponašanju

Učenje osnovnom ponašanju koje može uticati i na druge oblike ponašanja: motivacija i odgovor na više zahteva. Kada to savlada dete može nastaviti da uči druge oblike ponašanja.

lekovi – različitihospitalizacija: stalna – dnevna – privremena

Primarni rani autizam – prvi meseci životaSekundarni – deca čiji je razvoj bio normalan ili barem prividno normalan i javlja se 2-3 god. Nakon psihičkih povreda: odvajanja, smeštanja u ustanovu, porodičnih neprilika.

109. Terapija autizmaTerapija igrom, podrška roditeljima,dinamička psihoterapija, porodična terapija.kognitivno-bihevioralna terapija,

Bihevioralna terapija:najefikasnija u ranoj primeni (pre 4. god.) sa decom čiji je IQ najmanje 50;sistem nagrada-kazna;autistična deca slabo reaguju na sekundarne nagrade ili na socijalne nagrade;bihevior-modifikacija počinje psihološkom edukacijom roditelja. Izabere se ciljno ponašanje (obično jednostavno: imitacija, pravljenje jednostavnih zvukova, učenje jednostavnih znakova u komunikaciji);

26

Page 27: NEVENKA TADIC PDM - skripta

primarne nagrade imaju veću moć za autističnu decu.Teško je izvesti bih. terapiju jer ova deca teško opserviraju i imitiraju i teško komuniciraju.

110. Simbiotička psihozaOsnovna obeležja

Jaka strepnja i panika na odvajanje od znašajnih, detetu bliskih osoba, najcešće majke, koja služi kao kontrafobični objekat koji otklanja strepnju odvajanja.Naglašena dvojnost odnosa sa bliskim osobama. Dvojnost proizilazi iz snažne želje za sjedinjenjem sa dobrim delovima objekta i istovremene težnje da se izbegnu loši delovi. Nije došlo do potpunog sjedinjenja i povezivanja dobrih i loših delova u jedan celoviti objekat, tj. Ja deteta nije celovito i povezano.

Može da počne rano već krajem prve godine, ali najčešće se javlja izmedu 3.-6. godine, u edipnoj fazi, kod dece koja su se pre toga razvijala neupadljivo, ili su bila upadljivo razdražljiva.Bolest obično počinje nakon neke, makar i beznačajne povrede-odvajanja od bliskih osoba, medicinskih postupaka...Autistična deca su ravnodušna na prisustvo roditelja i ponašaju se kao da ih ne primećuju dok simbiotičko dete ne podnosi ni kratko odvajanje.

111. Meh. odbrane prepsihozama i prognoza i neurozePrepsihozama kao i neurozama svojstvena je raznorodnost i neusklađenost ja i njegovih odbrana. Za razliku od neurotičnih, prepsihotične odbrane primitivnije su i ponavljaju se.To su:

Projektivna identifikacija PoricanjePounutrenje Cepanje.Premeštanje

Strepnja, češće strepnja od uništenja, a ređe od kastracije, kod. prepsihoza, je ublja, replavljuje f-je ja, preti njegovoj celovitosti i povezanosti, pa su nova ulaganja otežana, što se ispoljava kroz smanjenje novih objektivnih veza.Intelektualne sposobnosti prepsihotišnog mogu biti dobre i pored slabog intelektualnog funkionisanja i neuspeha u školi.Prepsihotična deca ispoljavaju veliku spremnost da neprimetno odgovaraju na uobičajene okolnosti suprostavljanjem,povišenom agresivnošću i prelaskom na čin.Oko 2/3 rano psihotične dece ostaje ometeno u društvenom životu i nesposobno za samostalan život.Postoji mogućnost da neka prepsihotična deca kasnije ispoljavaju prividno neurotične kliničke slike.I ako dete uspostavi nekakvu ravnotežu, postoji mogućnost da se pod novim stresovima i sukobima, pri pokušaju uspostavljanja novih obj. odnosa i zbog teškoća novih ulaganja, ova ravnoteža poremeti, te da dođe do novih povremenih, ali i trajne psihotične razgradnje.Ako simptomi i odbrane ne uspevaju da nadziru duboku strepnju i depresivnost,Ako su kočenja jaka pa sužavaju granice ja,Ako ja nije u stanju da koristi nagonsku energijuPrognoza je loša

uzroci prepsihoza na dečijem uzrastuOrganski uzrociBiohemijski činiociGenetski uzrociPsihicki i društveno-kulturni činioci

112. U kom periodu su fiksirane osobe sa graničnim smetnjamau ranom uzrastu

27

Page 28: NEVENKA TADIC PDM - skripta

113. Normalno i patološko kod dece i mladih u poremećaju ponašanjaPotrebno je razlikovati stvaralačko prilagođavanje od nestvaralačkog, odnosno od potčinjavanja. Zato dobro ili loše prilagođavanje mora biti određeno, kako u odnosu na prilagođavanje prema sebi-autoplastično, tako i u odnosu na druge-autoplastično, da bi se procenilo njegovo pojedinačno psihološko i društveno značenje i vrednost.

114. Koje su odlike asocijalnog tipa1) Osećajne promene

Mržnja LjutnjaLjubomora TugaZavist MrzovoljnostDepresija Osećanje manje vrednostiNesigurnost Pojačana strepnja i drugi oblici strahovanjaBes Pojačano neprijateljstvo

2) Poremećaji obrazaca ponašanjaDurenje NeposlušnostSuprostavljanje TvrdoglavostSvadljivost NaglostBekstva TučLaganja

3) Smetnje saznajnih f-jaslabija pažnja inhibicijagubitak zanimanja za učenje mucanjintelektualna zakočenost

4) Preterani ili zakočeni pokreti5) Smetnje spavanja ,hranjenja, mokrenja, defeciranja, polnim zanimanjima idr.

115. Asocijalno ponašanjeNeposlušnost, fizičkim ili govorom ispoljeno neprijateljstvo, preosetljivost za greške drugih,

paranoidnost, sklonost ka nazadovanju, rušilaštvu, svadljivosti, suprostavljanju i pravljenju smicalica drugima i napadi besa.Teškoće opštenja sa drugima i stvaralačke saradnje, osvetoljubivost, sadizam, slab nadzor polnih težnji, okrutnost prema ljudima i životinjama, nanošenje neprijatnosti drugima, laganje, bekstva, skitanje, krađe, papkarenje, odsustvo osećanja krivice i stida, moralna manjkavost i izopačenost

116. Mladalačka krizaSmetnje prilagođenja u mladalačkom dobu,često može da otvori i put dubljim razgradnjama i patološkom razvoju,nema jedinstvenog načina ispoljavanja,može da poprimi obeležja svih drugih smetnji i poremećaja i biti uvod u druge poremećaje kao što su: neuroze, psihoze, psihopatije i nedruštvena ponašanja,nagle telesne promene,smanjenje intelektualnih i drugih zanimanja, intelektualna kočenja, slab uspeh u školipovećanje mišićne mase,promena oblika i izgleda delova tela,fiziološke, hormonske promene,

28

Page 29: NEVENKA TADIC PDM - skripta

pojava polnih oznaka,razvoj apstraktnog mišljenja dostiže viši stepen u ovom uzrastu,promena stavova prema mladoj osobi, roditelji isto doživljavaju krizu,menjaju se odnosi sa roditeljima: postaju labavi, lomljivi, često ispunjeni dvojbom i neprijateljstvom,osećajni život mlade osobe je značajno izmenjen : osećajna zbrka sazdana od protivrečnih osećanja ljubavi i mržnje, tuge, depresije, strepnje od odvajanja.

117. Intelektualizacija po Ani Frojd i ciljAna Frojd primećuje da su mladalačkom dobu svojstveni intelektualizacija i asketizam kao načini odbrana od naraslih nagonskih težnji i straha.Intelektualizacija svedoči o pokušaju da se upravlja naraslim nagonskim težnjama vezujući ih za, i omladincu i društvu, prihvatljive misli.Kada je indikovana psihoterapija kod hiperkinetičkog sindroma, indikovana je kada se sumnja na postojanje unutrašnjeg konflikta koji leži u osnovi poremećaja.

118. Kriza identiteta Je prekid u dotada igzrađenom telesnom, polnom, osećajnom i društvenom identitetu. Sastavni je deo mladalačke krize. Mlada osoba se ponaša kao da ne zna ko je i šta je, da li je ona ona ista

ili neka nova. Prekidaju se poistovećivanja sa drugima Usamljenost Povišena narcistička ulaganja Postavljanje pitanja o sebi... Počinje u pubertetu ili nakon njega u ranijoj, ređe u kasnijoj mladosti, do 25. godine. Tok zavisi od prethodnog razvoja sklopova ličnosti, odnosa između stepena unutrašnje povezanosti i

stavova okoline. Mogu predstavljati plodnu fazu i biti mogući organizator daljeg razvoja.

119. Koje su najčešće potrebe u pozadini dečijih krađaKod mlađe dece sam čin prisvajanja može da zadovolji potrebe za posedovanjem i da ima f-ju rasterećenja od napetosti i strepnje, f-ju traženja kazne, pribavljanje ljubavi od roditelja, nadoknada za odbacivanje, suparničenje sa povlašćenim detetom.Ukradeni objekt može da bude koristan, ali često ima simboličko značenje-ljubav roditelja, prestiž nad drugom decom...Malo dete može da veruje da je ukradeni objekat deo voljene osobe.Predmet se ili koristi ili poseduje, ali ga dete često razara, krije i poklanja.U fazi mirovanja, naročito kasnije u mladosti, krađa je sve ređi izraz neprilagođenosti i neurotičnosti, a sve češće izraz psihopatskih obeležja ličnosti.

120. Šta je mitomanija?Laganje - neistinito prikazivanje stvarnosti. Podela po A. Frojd:

1) Nedužno laganje, 2) Mitomanstvo, 3) Prestupničko laganje

1) Mlađe dete zbog nedovoljno razvijenih saznajnih f-ja i nedovoljnog razlikovanja realnog od nerealnog između primarnog i sekundarnog procesa po pravilu netačno prikazuje realnost. Njegov doživljaj događaja ili izmišljena priča njemu su istinita i on ih saopštava naivno i dobronamerno kao nedužnu laž.

Na starijem uzrastu dete počinje da razlikuje stvarno od nestvarnog i uskoro njegova istina postaje realnija i opštija

29

Page 30: NEVENKA TADIC PDM - skripta

2) Zbog snažnih osujećenja u stvarnom svetu i sa željom da se zadovoljenja nadoknade, jedna grupa dece, mladih pa i odraslih nadopunjuje nepoželjna iskustva i doživljaje izmišljenim i za njega uvek poželjnim sadržajima.

Ovi mitomanski sadržaji treba da ispune želje i očekivanja i nadoknade razočaranja i osujećsnja.1) Laganje s predumišljajem da se dobije nešto ili izbegne kazna, posledica

zove se prestupničko laganje.Javlja se kod : konverzija, društvenih poremećaja i psihopatija.

121. Paljevine kao poremećajDeca se rano igraju vatrom i paljenjem i pri tome doživljavaju uzbuđenje i radost. Plamen deluje uzbiđujuće na čulo vida i kožu. Međutim, svaka igra i paljenje ne mogu biti uvštene u nedruštveno ponašanje. Jave se i kao normalna igra u detinjstvu, koja podstiče uzbuđenja i maštu, kao agresivna radnja, prkos i otpor roditeljima i školi, način kažnjavanja drugih, prisilni simptom u okviru prisilne neuroze ili prisilnog karaktera.Za starije dečake-vatra i paljenje-polno značenje.Sem toga vatra i paljenje imaju simboličko značenje - žežu, kažnjavanju, razaraju, čiste dobro od zla.Paljevine češće obavljaju dečaci starijeg uzrasta čiji je osećajni i saznajni razvoj usporen, niže inteligencije, moralno bezbrižni i nemaštoviti.Roditelji piromana su najčešće : odbacujući, osujećujući, podstiču razaračke radnje.

122. Četiri bihevioralne tehnike u radu sa decom i mladima sa poremećajem ponašanjaUčenje po modelu, Uvođenje deteta u rešavanje problema,Nagrađivanje prosocijalnog ponašanja, Psihoedukacija,Time-out, Nadzor od strane drugih osoba,

Korišćenje jednostavnih naredbi.

123. Nabroj uzroke psihopatskih oboljenjaPoremećaj ličnosti -- psihopatijeR. Špic - već u 2. godini života uzrok - grubo izmenjeni objektni odnosi između deteta i roditelja.Deca su u porodici izložena jakim i dugotrajnim, osujecenjima,odbaćivanju zanemarenju, neprijateljstvu, grubom kažnjavanju, nedoslednim promenljivim i nepredvidivim stavovima roditelja.Neka deca su odrasla u preterano popustljivim porodicama bez ikakvih ograničenja ili prepuštena sama sebi i uličnim grupama. Dete nije imalo primerene uzore za oponašanje i poistovećenje.Neki autori govore o psihopatijama pretežno neurotičnog tipa. Tada su radnje praćene naknadnim osećanjem krivice, iako je osećanje krivice za vreme radnje prividno odsutno.Prvobitna i naknadna agresija roditelja, koja izazivaju ponašanja deteta i omladinca imaju značajnu ulogu.

30

Page 31: NEVENKA TADIC PDM - skripta

124. Karakteristike i psihodinamika dece sa astmomTokom razvoja na regulaciju disanja i na rad bronhijalnih mišića utiči osećanja. Ritam i jačina disanja menjaju se u stanjima uzbuđenja, straha, osujećenja, besa... Bronhijalna astma najčešće psihosomatsko oboljenje dece i omladine

Psihodinamika:Napadi sa promenama u ritmu udisaja i izdisaja zbog grča bronhiola i poremećaja u lučenju sluzokože disajnih organa.Praćeno jakim doživljavanjem straha od ugušenja. Slično osećanje doživljavaju i roditelji.

Karakteristike:Tip ličnosti :Povezana strepnja i zavisnostNepoverljivost i napetostSlabije ustrojstvo jaPrezaštićujuća majkaVisoka očekivanja od detetaTip roditelja : prezaštićujuće ili bojažljive ,visoka očekivanja od deteta, nagle, hladne, depresivne majka.

125. Alergijski tip ličnostiKrajsler smatra da asmatična deca pripadaju alergijskom tipu ličnosti :Potiskivanje agresivnostiPoricanje sukobaPreterano korišćenje pomeranja i povredljivostNe ispoljavaju strah od nepoznatih osobaStvaranje alergijske ličnosti dovodi se u vezu, kako sa urođenim psihičkim, tako i sa urođenim alergijskim činiocima.

126. Krajsler- sklopovi ličnosti podložni psihosomatskim oboljenjima1. Neuroze ponašanja2. Nedovoljno organizovane ličnosti3. Depresivna deca-omladina4. Alergijski sklop ličnostiPsihosomatski poremećaj posledica je aleksitimije.

127. Šta se pre javlja: psihosomatski poremećaj ili neurozaPsihosomatski poremećaji su stariji od neurotičnih jer se javljaju u ranom detinjstvu pre stvaranja neurotičnog sukoba između delova ličnosti.Psihosomatski poremećaj je neposredni izraz afektivnog trpljenja koji u početku menja funkciju, a zatim i sklop organa.

128. Šta je mericizam - psihosomatski poremećaj probavnih organa Preživanje hrane ili mericizam Funkcionalni poremećaj vraćanja progutane hrane u usta, prežvakavanja i ponovnog gutanja Nije česta pojava Javlja se kada je dete ostavljeno samo sebi Udruženo je sa kompulzivnim autoerotskim radnjama, klaćenjem glavom i sisanjem. Posledica je odbacivanja od strane roditelja, agresivnosti i manjkavih odnosa sa objektima, kao i

nedostatka drugih podržaja. Vraćajući i prežvakavajući već jednom progutanu hranu, dete zadovoljava glad i pohlepu za odnosom sa

objektima, obezbeđuje zadovoljstvo kroz radnje ustima i autoerotsko zadovoljenje. Mericizam je češći kod dečaka.

31

Page 32: NEVENKA TADIC PDM - skripta

129. Prilagođeni i neprilagođeni debiliUmni razvoj – ne prelazi uzrast deteta od 10. godinaIQ – kreće se od 50 do 70

Prilagođeni – skladni:

ne ispoljavaju veća odstupanja u odnosu na osobe normalne inteligencijeuspevaju da koriste svoje sposobnosti za sticanje znanja, da se srazmerno svojim intelektualnim sposobnostima dobro prilagođavaju porodici i široj sredini i da se odgovarajuće profesionalno osposobe.Neprilagođeni – neskladni:ispoljavaju značajne osećajne smetnje i taškoće društvenog prilagođavanjaviđaju se apatični, uznemireni, nepostojani, detinjasti, osećajni, manični, dr. tipoviego je slabiji, lomljivmehanizmi odbrane su primitivnijipoistovećenja su češće zbrkana, zbrka između želja i doživljenog je očigledna, procena stvarnosti je nepotpunapovišena agresivnost : neprihvaćenost od okoline

130. Poremećaj sa najgorom prognozomDemencija, najčešće je konačno stanje sa malim izgledima za poboljšanje. Bolja prognoza kod demencije nastale zapaljivim procesima (encefalitis), jer može da dođe do obnavljanja tkiva. Loša prognoza demencije koje su nastale heredodegenerativnim oboljenjem CNSa i kod dece i omladine koji su pokazivali osećajne promene i promene u druš. Ponašanju.

131. Koji poremećaji su češći kod slepe nego kod zdrave dece(zašto)Kod slabovidih ispitanika: depresivne težnje, strepnja, nevoljno umokravanje (6%).Među slepim ispitanicima: fobične težnje, konerzivno-fobične težnje, nevoljno umokravanje (10%).

Opšta i posebna neurotičnost slepih i slabovidih ima ista obeležja kao i neurotičnost dece i omladine koja vide.Organizovane i ustaljene neuroze, kao i psihoze slepih i slabovidih isto toliko su česte, imaju ista obeležja, uzroke i dinamiku kao i među decom i omladinom koja vidi.Sa polaskom u školu slepa i slabovida deca mogu da ispolje teškoće prilagođavanja zbog pojačanih zahteva škole, ulaska u grupu drugova i povećane svesti o svom nedostatku.

132. Terapija zlostavljane decePodrazumeva kako terapiju roditelja tako i terapiju deteta. Terapija koja će se primeniti zavisiće od vrste zlostavljanja:Fizičko, Seksualno,Emocionalno, Zanemarivanje.

Treba ispitati gde je dete zlostavljano, u porodici ili izvan nje,Treba videti koji je član porodice u pitanju.Ostali treba da pruže podršku i zaštitu detetu i umanje osećaj krivice.Terapija ohrabrivanja.Terapija igrom.Kognitivna terapija – trebalo bi da pomognu detetu da stekne uvid da nije krivo, već da je greška osobe koja zlostavlja.

133. Psihoterapija za kontrolu besa

32

Page 33: NEVENKA TADIC PDM - skripta

Socijalna – vežbanje socijalnih veština, Rešavanje problema,Psihološka, Anger management (vođenje/kontrola besa),Medikamentozna – neuroleptici, antidepresivi, Igranjeuloga.

134. Tehnike kod besa (10 koraka)Bihejvioralne intervencije

Psihoterapija (sve iz 205 pitanja), Neusmerena (spontana) igra,Posmatranje i nadzor odraslih, Korišćenjejednostavnih naredbiOhrabrivanje društveno prihvatljivog ponašanja, Učenje po modelu (video trakama),Time – out (kazna, uskraćenje, izolacija), Synthesis tehnika,Uključivanje deteta u rešavanje problema, Vežbanje samodovoljnosti.

135. Vilijamsov sindromRetka genetska bolest okarakterisana slabošću za obuzdavanje, ublažavanje mentalne retardacije teškoće učenja. Karakterističan izraz lica uključuje natečenost u predelu očiju, kratak nos, široka usta, pune usne i malu bradu. Osobe sa VS mogu takođe, da imaju dug vrat, spuštena ramena, nizak rast, pgraničenu pokretljivost zglobova i pokretljivost kičeme.Više od 50 % dece sa VS imaju deficit pažnje, oko 50 % specifične fobije, koje se odnose na strah i jake zvuke.VS je nedovoljno poznato i komplexno oboljenje. Lečenje se bazira na individualnim simptomima.Prognoza je individualna.

136. Aspergerov sindromJavlja se:

oštećenje recipročnih socijalnih interakcija, koje karakterišu autizam, uz skučene, stereotipne, repetitivne repertoare, interesovanja i aktivnosti.Većinom se javlja kod dečaka.Razlika od autizma = ne postoji opošte kočenje i retardacija govora ili kognitivnog razvoja.Najveći broj individua je normalnih intelektualnih sposobnosti, ali su često trapavi.Javlja se obično u 3. godini.Moguć i visok IQ.Blaža forma autizma. - Oštećena desna hemisfera.

137. Koji su najčešći fantazmi u dečijim strahovima1) najveći br. strahova odnosi se na osobe za koje je dete najviše vezano;2) brige u vezi sa mogućom povredom osobe za koju je dete najviše vezano;3) strah da će osobe otići i neće se vratiti;4) briga da se ga neki događaj, npr. odlazak u bolnicu ili da se izgubi, odvojiti od osobe;5) strah da ostane samo ili bez osobe za koju je najviše vezano.

138. Uzroci posttraumatskog stres sindroma

33

Page 34: NEVENKA TADIC PDM - skripta

Karakteristika PTSS je razvoj simptoma koji smetaju osobi, ali se mogu izbeći, a javljaju se nakon uzlaganju nekom traumatičnom događaju. Traumatičan događaj se definiše pomoću:

1. Izlaganja smrtnoj opoasnosti, tj. Ugrožavanju fizičkog integriteta ličnosti. Osoba ne mora biti učesnik u događaju, mada je potrebno određeno lično iskustvo, npr. Posmatranje traumatičnog događaja

2. Doživljeno iskustvo izaziva intezivan strah, užas, bespomoćnost, dezorganizovana ponašanja ili suzdržana. Poslednja dva se javljaju samo kod dece.

Simptomi su uglavnom isti kod odraslih i dece, s tim da se kod odraslih više osećaju na kognitivnom planu, a kod dece na bihejvioralnom.Da bi se postavila dijagnoza potrebno je da simptomi postoje bar 1 mesec. Ako se simptomi jave odmah posle nesreće 2 dana-4 nedelja, radio se o akutnom posttraumatskom sindromu.

Lečenje: bihejvioralne tehnike – desenzibilizacija; psihoterapija – terapija fokusiranja na traumu.

139. Koje su bolesti češće kod dečaka a koje kod devojčicaDečaci: Devojčice:

1) somnabulizam, 1) mentalna anorexija,2) noćno mokrenje. 2) čupanje dlaka,3) defeciranje, 3) samoubistvo,4) samoubistvo uopšte, 4) neuroza (12 – 19 god.)5) lupanje glavom, 5) fobična neuroza.6) tikovi,7) hiperkinetika,8) porem. ponašanja,9) toxikomanija,10) umna zaostalost,11) neuroza (7 - 11),12) Aspergerov sindrom.

34

Page 35: NEVENKA TADIC PDM - skripta

35

Page 36: NEVENKA TADIC PDM - skripta

PDG

1. Intermitentnost – alternativnostIntermitentnost - neki oblici ponašanja mogu da se pojave u određenom uzrastu, a zatim se '' izgube'' i posle izvesnog vremena opet pojave. Intervali ''gubljenja'' se skraćuju sve dok se ponašanje potpuno ne učvrsti. (npr. Dete koje je prohodalo može povremeno da se vrati na puzanje)

Alternativnost - pojava da se u nekim periodima neke funkcije ili grupe funkcija razvijaju brže i više, '' na račun '' drugih, koje privremeno stagniraju. (Buran razvoj motorike nasuprot relativno sporom razvoju govora u 1 i 2 god.)

2. Regresija u službi jaU dečijem uzrastu regresija najčešće predstavlja ''korak nazad radi dva napred'' tojest nazadovanje u trenutno preteškoj situaciji da bi se sakupile snage za dalju progresiju. Najčešće je koristan činilac u procesu razvoja. Njeni učinci su spontano reverzibilni.U dečijem uzrastu je česta, lako prepoznatljiva, ima korisne i socijalno prihvaćene modalitete (igra).– K r i s –

3. Nabrojte 3 organizatora psihe po ŠpicuSvaki stupanj razvoja ima jedan organizujući proces koji integriše činioce sazrevanja i psihičkog razvoja koje Špic naziva organizatorima psihe preko kojih pokušava da opiše proces dostizanja novih stupnjeva integracije.1) Prvi znaci da dete postaje svesno konfiguracije ljudskog lika dobijaju potvrdu kroz *socijalni

osmeh* trećeg meseca koji predstavlja znak selektivnog reagovanja na spoljašnji podsticaj i početak formiranja Ja te ga Špic naziva prvim organizatorom psihe.

2) Drugi organizator psihe *strah od odvajanja* (anaklitički strah) predstavlja najavu uspostavljanja libidnog objekta.

3) Treći organizator psihe *usvajanje i korišćenje semantičkog znaka ne* rečima i gestovima, i upotreba reči u simboličnom smislu, kao prihvaćenih zahteva društvene sredine. Ja je dostiglo kapacitet za apstrakciju i reverzibilnost.

Prvi organizator – struktuira opažanje i postavlja osnove Ja. Drugi organizator psihe – integriše objektne odnose sa nagonima i uspostavlja Ja kao organizovanu psihičku strukturu sa različitim sistemima. Treći organizator psihe – stvara put za razvoj objektnih odnosa uz pomoć semantičke komunikacije.

4. Pet nekonflintnih funkcija poFrojduOpažanje – na šta ukazuje mogućnost razlikovanja voljenog objekta od svih ostalih.Voljne aktivnosti – omogućene dovoljno razvijenim motornim sposobnostima.Rasuđivanje – koje je omogućeno postojećim tragovima pamćenja.Ranog razvoja načela realnosti – na šta ukazuje početna poslušnost zabranama i pravilima ponašanja.Sintetička funkcija ja – obezbeđuje osnovu za integraciju dobrog i zlog objekta, odnosno fuziju (i neutralizaciju) libida i agresivnosti.

36

Page 37: NEVENKA TADIC PDM - skripta

5. Kastracioni strah kod dečaka i devojčicaDečak:

kastracioni strah mi omogućava izlaz iz edipalne situacijestapanje libidnih i reaktivnih zežnji pod pretnjom kastracionog straha, odnosno pod uticajem stvaranja moralnih normi pretvara tako edipalnu privlačnost prema majci u sinovljevu ljubav.

Devojčice:kastracioni strah dovodi do edipovog komplexa.razrešenje edipovog komplexa je složenije . Da bi ga rešila, ona po drugi put mora da napusti objekat ljubavi i želje (sada oca) i da se vrati prvobitnom objektu (majci). Ovo je moguće ako prihvati činjenicu anatomije sopstvenog pola i iskoristi suštinski sadržaj komplexa kastracije koji „inhibira i ograničava mupkost,ali obećava ženskost“.postepeno napuštanje E.K. putem potiskivanja će kod nje u velikoj meri da zavisi od nasleđivanja o materinstvu, ženskosti...39.Edipov kompleks i kastracioni strah

Na uzrastu izneđu 3. i 6. godine dete prolazi kroz niz promena i usložnjavanja odnosa prema objektima i prema sebi. Fantazam o odsečenom, uništenom, i izgubljenom penisu čini suštinu fantazma kastracije. Razliku među polovima dete ne shvata kao objektivno anatomsku razliku između telene građe muškarca i žene, već pod dominacijom prvog falusnog primata koji daje 2 mogućnosti – posedovati penis ili biti kastriran. Kastracioni strah rezujtat je pogrešne predstave o realnosti, on je razvojni fenomen. Roditelje dete smatra odgovornima za svoje stanje i u njih projektuje sadističke (agresivne) težnje, mada prolazi i kroz period kom i dalje veruje u falusnu majku, koja kao idealizovana osoba ne podleže kastraciji. Strah se može povećati zabranama, pretnjama kaznama koje roditelji usmeravaju na dete, kojim reaguju na pitanja deteta koji su izraz radoznalosti da se sazna istina o razlici među polovima, o sebi, o svom rođenju, nastanku.

Kastrativne intervencije uvećavaju neizbežnu i normalnu strepnju deteta braneći se od nje, dete ulazi u edipalni odnos. Libidna investiranost polnog organa, saznanje da i mama i tata pripadaju njegovom, odnosno onom drugom polu, potreba za posredovanjem, zavist i destruktivnost, pored agresivnosti i pasivnosti – pripremili su teren za doživljavanje edipalnog odnosa. Ono se razlikuje kod devojčica i dečaka. Kastracioni strah onemogućava dečaku izlaz iz edipalne situacije. Postupno razrešenje edipnog sukoba oslobađa u svesnom kao i nesvesnom planu znatnu količinu libidne energije, koja može biti sada investirana u nove objekte i nove aktivnosti.

Kod devojčica kastracioni strah dovodi do E. kompleksa, kod njih kompleks kastracije omogućava i najavljuje edipov kompleks. POstepeno nestajanje E.K. putem potiskivanja će u velikoj meri zavisiti od naslućivanja o materinstvu, ženstvenosti itd. U mnogome zavisi od ponašanja majke i stavova prema sopstvenom polu, jer će to posredno omogućiti devojčici da se oslobodi osećanja krivice zbog incestuidnih žečja i stvori teren za libidno ulaganje ženskih erogenih zona.

6. Razvojni strahovi, kada se javljaju?Ne nastaju usled spolj. ugroženosti, već usled nezadovoljenih nagonskih težnji, odnosno siline

instiktivnih pokretača i sastavni su deo psihičkog razvoja. Javljaju se kao posledica – biološke i psihološke nezrelosti odnosno zavisnosti od majke, odraslijh.

Razvojni strahovi su:Strah od komadanja (prvih 6 meseci)Strah od odvajanja (8. Mesec -16....)Strah od prljanja (2. Godina)Kastracioni strah (edipalni period 3 -5 godina)Strah od kazne nad – ja (oko 5 -6 godine)

37

Page 38: NEVENKA TADIC PDM - skripta

1.) Strah od komadanjaU prvoj polovini prve godine usled pretnje mogućeg gubitka kontrole nagonskih težnji,

odnosno gubitka tek lagano uspostavljenog integriteta ja, u situaciji kada još nisu uspostavljene telesne i mentalne granice ja, a investiranost psihičkom energijom je još na nivou parcijalnih objekata.

2. )Strah od odvajanjaSeparacioni / anaklitički stres oko 8. meseca detetovog života, u vreme kada majku počinje da

doživljav i prepoznaje kao zasebnu ličnost. Traje do oko 16 –tog meseca kada dete ''izrasta'' iz ovog straha, koji ostaje manje/više prepooznatljivi ko sastavni deo čovekovog emocionalnog života.

U dečijem uzrastu on se sasvim lako vezuje za kaznu zbog realno učinjene ili samo zamišljene neposlušnosti, prestupa, načinjene štete.

3.) Strah od prljanjaRazvojno je vezan za drugu godinu života kada preko poistovećenja sa roditeljskim

zabranama dete razvija mogućnosti pretvaranja prvobitne nagonske potrebe za prljanjem u njenu suprotnost.

4.) Kastracioni strahStrah edipalnog perioda razvoja odnosa prema objektu i u raznim varijantama poprima vid

strašljivog iščekivanja kazne zbog nekog nedozvoljenog ponašanja sopstvene agresije, nedozvoljenih nežnih osećanja, ljubomore...

5.) Strah od kzne nad - jaOko 5 – 6 godine, u vreme kada je već formirana ova instanca ličnosti. Može da bude veoma

jak kada je dete prekršilo etičke odnosno moralne norme koje je usvoilo putem poistovećenja sa roditeljima. Nastaju iz nezadovoljenih težnji.

Deca se tokom vremena sukobljavaju sa izvesnim situacijama u kojima prepoznaju svoje razvojne strahove u raznim varijantama.

Po svom karakteru i strukturi srodni su fobičnom stanju odraslog, jer nastaju u vreme pre nastanka genitalnog primata, pa zato nisu u stanju da se organizuju i utope u sve one načine odbrane koji stoje na raspoloženju zreloj ličnosti.

7. Prisila ponavljanjaPoznati način dečijeg prevazilaženja straha je tzv. prisila ponavljanja. Sastoji se u tome što kroz više puta ponovljenu situaciju koja zastrašuje - a deca to čine kroz igru, crtež, priče, šalu – dete se upoznaje sa zastrašujućom situacijom, predmetom ili osobom, snaga strašnog se smanjuje, dete „manipuliše“ strašnim time što ono ponavlja situaciju i tako postepeno ovladava strahom i prevazilazi ga.

8. Kada se dete igra i u igri oživljava sukobe taj mehanizam odbrane je:Prisila ponavljanja

9. Identifikacija(poistovećenje) kao razvojna pojava i kao mehanizam odbraneRazlikuje se od ostalih mehanizama odnosno približava sublimaciji.Predstavlja normalan razvoj ni proces kojim se ličnost trajno menja i obogaćuje.To je kontinuiran proces koji podrazumeva pomeranja ciljeva nagonskih pokretača i određeni utrošak psihičke energije. Biti kaao objekt idetntifikacije znači veću mogućnost da se poseduje željeni objekat. Dete se privremeno odriče želje da poseduje objekat u korist želje da postane

38

Page 39: NEVENKA TADIC PDM - skripta

slično objektu poistovećenja. Ovo pomeranje cilja instinktivnog pokretača omogućava znatno energetsko rasterećenje.

Kao meh. odbrane u užem smislu te reči javlja se u različitim modalitetima, među kojima su najpoznatiji:

1) poistovećenje sa voljenim objektom, ličnost se formira putem usvajanja karakteristika voljene osobe, što smanjuje strah od odvajanja i napetost koju izaziva ambivalencija.

2) poistovećenje sa izgubljenim objektom, preuzimaju se karakteristike nekog ili nečeg da bi se umanjio ili poništio gubitak.

3) poistovećenje sa agresorom, predstavlja *manevr* izbegavanja strepnje na taj način što se postaje sličan zastrašujućem objektu, a aggresivnost se doživljava kao da potiču iz same osobe, koja se poistovećuje, i na taj način može kontrolisati.Da bi se zaštitilo od osobe koja plaši i izbeglo strah od njene agresije, dete se poistovećuje sa agresorom i tako se i ponaša (bije lutke, drugove). Dobro i dosledno poistovećenje sa lošim likovima ima ulogu u pojavi nedruštvenih ponašanja. Ako se to desi od prvih dana nedruštveno ponašanje postaje jedini način ponašanja. – Ana Frojd

4) poistovećenje usled osećanja krivice, je samokažnjavajuće poistovećenje koje je posledica ambivalentne investicije objekta, pri čemu kažnjavajući sebe osoba uživa u kazni.

10. Ko uvodi u psihoanalizu aktivno – pasivno?Frojd je široko koristio ideju postojanja polariteta između aktivnosti i pasivnostiPo klasičnom psihoanalitičkom tumačenju:

Aktivnost se odnosila na Pasivnost naMaskulinost FemininostAgresiju SubmisivnostSadizam MazohizamVoajerizam Egzibicionizam

Ova podela se komplikovala daljom pretpostavkom da instinkti mogu da pretrpe obrtanje u suprotno, naročito aktivni instinkti mogu da postanu pasivni, pri čemu su se kao primer najčešće izdvajali sadizam i voajerizam. U kasnijem razvoju psihoanalize jednačina aktivno – maskulini, a naročito pasivno – feminino, pretrpela je niz izmena.

11. Autoplastična i aloplastična adaptacijaTermine uveo F. Aleksander 1930 godine da bi podvukao nejednakost između adaptivnih

odgovora koji menjaju sredinu - aloplastično i onih koji menjaju self - autoplastično

Mehanizmi odbrane psihoneuroza su klasični primeri autoplastične adaptacije.

12. Ego - sintono i ego - distonoZa ponašanje i želje kažemo da su ego–sintone kada su saglasne sa subjektovim idealima ili shvatanjima o samome sebi.Za ponašanje i želje kažemo da su ego–distone kada nisu saglasne sa subjektovim idealima ili shvatanjima o samome sebi tj. odnose se na vrednosni sud koji donosi sam subjekt.

39

Page 40: NEVENKA TADIC PDM - skripta

13. Zbog čega se javlja fiksacija?Proces putem koga osoba postaje/ostaje ambivalentno vezana za neki objekat, pri čemu je taj

objekat bio odgovarajući ranijem stupnju razvoja.Fiksacija - znak nedovoljno uspešnog razvoja kroz faze razvoja odnosa prema objektu.

Osoba sa fiksacijom:Ima tendenciju da pribegava infantilnim šemama ponašanja ili da nazaduje na ove vidove ponašanja kada je pod stresom.Da prisilno bira objekte na osnovu njihove sličnosti sa objektom za koji je fiksirana.Trpi zbog nedostatka raspoložive psihičke energije usled njene uloženosti u taj objekat.

Preterana osujećenja i zadovoljenja, preterana ljubav i mržnja smatraju se mogućim uzrocima fiksacije.

14. Pojam inkorporacijePonekad se koristi kao sinonim za internalizaciju ili introjekciju, odnosi se samo na fantazam

unošenja spoljašnjeg objekta. Fantazmi inkorporacije su tipično oralni (gutanje objekata), ali mogu da se odnose i na druge otvore tela (oko, uvo, čula). Zamišljeno uvlačenje u telo kroz otvore (usta, čula) delove spolj. objekata (dojke, mleko).

15. NeutralizacijaPosledica fuzije kojom određene težnje postaju sposobne da budu prihvaćene od svesnog ega

i izvedene preko mišića, čula ili sposobne da se pojave u svesti.Ukoliko težnje nisu neutralisane samom fuzijom one bvaju obrađene mehanizmima odbrane koji dovršavaju neutralizaciju

16. RestitucijaOdbrambeni proces smanjivanja krivice iskupljivanjem prema ambivalentno investiranom objektu.Proces kojim shizofreni ili paranoidni pacijenti stvaraju sumanutosti koje im omogućavaju vraćanje osećanja značaja.

17. ReparacijaProces (mehanizma odbrane) putem koga se smanjuje osećanje krivice tako što se u dobro

pretvara ono što se zamišlja da je loše učinjeno ambivalentno investiranom objektu.Proces obnavljanja unutrašnjeg objekta koji je bio razoren u fantaziji.U klajnijanskoj teoriji - sve kreativne aktivnosti se smatraju reparativnim, u skladu sa

stanovištem da reparacija predstavlja normalan proces putem koga jedinka razrešava inherentnu ambivalenciju prema objektima.

40

Page 41: NEVENKA TADIC PDM - skripta

SMIRNOF

1. Konstitucionalna datostJe izvesno stanje mentalnig funkcionisanja koje osposobljava osobu, više-manje, da u slučaju

potrebe izdrži frustracije, stres i da integriše tokove sazrevanja.

2. Šta je konstitucionalnoPodrazumeva sve urođene faktore koje dete donosi sa sobom na rođenju. Oni obuhvataju kako nasledne, tako i stečene činioce u toku intrauterinog života. Zavise od fiziološkog i psikološkog stanja majke.

3. Rakamije – od čega zavise posledice ranog afektivnog lišavanjaPosledice su najozbiljnije na psihološkom planu. Osećajni nedostatak može da prouzrokuje zastoj razvoja koga treba razlikovati od pravih mentalnih zaostalosti. Takvo stečeno nesazrevanje svedoči o deficitu JA u njegovoj f-ji posrednika između osobe i okoline.Osoba ima nesposobnost da voli i da bude voljena.Rakamije je to označio kao apersonalizaciju i što objašnjava teškoću koju imaju takva deca da se identifikuju sa roditeljskom slikom.Postoje odojčad koja su osetljivija nego druga deca na afektivni nedostatak. Rakamije to objašnjava nivoom instinktivno – vegetativne integracije, koja je konstitucionalni činilac = neka odojčad imaju naročito nizak prag na frustraciju.

Posledice af. liš. su značajnije ukoliko nailaze ranije u životu;Posledice su jače ukoliko osećajno liš. traje duže;Rano odvajanje stvara nepovratne posledice.

4. Rakamije – činioci afektivnog lišavanjaPosledice su najozbiljnije na psihološkom planu, osećajni nedostatak može prouzrokovati zastoj razvoja.Osiromašenje JA, koliko u narcističnim, toliko u objektnim investiranjima – nesposobnost da se voli, da se bude voljen – Rakamije je označio nazivom apersonmalizacija i što objašnjava teškoću koju imaju takva deca da se identifikuju sa roditeljskom slikom. Javljanje i razvoj sindroma afektivnog nedostatka podleže izvesnim činiocima:

1) postoje odojčad koja su osetljivija nego druga deca na afektivni nedostatak. Rakamije to objašnjava - neka odojčad imaju naročito nizak prag tolerancije na frustraciju;

2) posledice su u toliko značajnije ukoliko nailaze kasnije u životu deteta. Najveće su u toku 2. i 3. tromesečja i utoliko su jače ukoliko je odojče od početka bilo podizano u boljim uslovima;

3) posledice su utoliko jače ukoliko traju duže;

41

Page 42: NEVENKA TADIC PDM - skripta

4) materinstvo, pod raznim vidovima, omogućava obnavljanje normalnog razvoja. Ali nakon izvesnog trajanja isuviše jake jačine, rano odvajanje stvara nepovratne posledice koje mogu biti ravne konačnom defektu budući da je deficit utisnut u samu strukturu psihosomatske organizacije osobe.

5. Po Frojdu – odnos psihičkog traumatizma i neurozaRođenje kao osnovni raskid veze sa majkom, odbijanje od dojke kao prelaz od potpune dečje zavisnosti do prvog stupnja nezavisnosti, Edip kao napuštanje objekta prve želje – bitni su za ličnost i neurozu. Ali ukoliko naknadne okolnosti ožive afekt vezan za ove traumatizme koji su stvorili nerešena i potisnuta konfliktna neraspoloženja, oni će u buduće moći da igraju patogenu ulogu.

Majka koja patogeno zaštićuje,Afektivno odsutan otac,Prikriveni sado-mazohistički odnosi roditeljskog para...

mogu najverovatnije biti patogeniji nego traumatični događaji koji se rado optužuju da bi se objasnile neuroze.

6. Ekran sećanjaAmnezija obuhvata sve bitne događaje ranih konfliktnih situacija ako bi ovi, zbog afektivne nezrelosti deteta, bili suviše mučni da bi im dete pristupilo, suviše teški da bi bili rešeni, te bi jedino poništenje omogućilo detetu da ih zaboravi umesto da ih reši.Tako cele površine detetovog života propadaju u zaborav, dozvoljavajući često da prežive samo indiferentna sećanja, ekran sećanja, mnestički predstavnici lišeni svakog efekta

7. Tri fantazma po Frojdu1) Fantazam falusne majke,2) Fantazam kastracije,3) Fantazam roditeljskog koitusa (Frojd ga je nazvao „primitivna scena“).

8. Retroaktivno poništavanjeŽeli da se učini ili misli nešto što bi bilo određeno da izbriše, na realan ili imaginaran način, akciju ili misao koju JA ne prihvata.

9. Polimorfno kod Frojda

Uobičajeno je govoriti o polnim nastranostima u odraslom dobu.

Kako je dete i normalno „polimorfno“ nastrano od rođenja (Frojd) i njegova polnost podvrgnuta igri „parcijalna pulzija“ povezanih sa drugim delovima tela, a ne genitalnim, izraz polna nastranost retko se koristi u dečijoj psihijatriji sem izuzetno, kad su odstupanja u psihosexualnom razvoju upadljivo velika i konačna.

10. Razlike između klasične psihoanalize i teorije objektnih odnosaSa pojmom objektni odnos analitička psihoanaliza je težila da objasni interakciju osobe i okolne sredine - kako preko postupnih faza razvoja osoba malo po malo uspostavlja svoje veze sa objektima, tj. kako se odvija prelaz od fiziološkog ka psihološkom, od materičnon parazitizma do ustanovljenja hijerarhizovanog društvenog odnosa.Kod psihoanalize izraz objek je ono što obezbeđuje zadovoljenje pulzije. Po Frojdu objekt nije nužno spoljašnji i može da sačinjava deo vlastitog tela.

42

Page 43: NEVENKA TADIC PDM - skripta

11. Seksualne pulzijeRazlikuju se od drugih instinktivnih pulzija po sledećim osnovnim crtama:

opire se prilagođavanju osobe na spoljašnje uslove;suprotno gladi ili žeđi sexualne pulzije su pogodne za auto-erotsku gratifikaciju i mogu biti

čak trenutno zadovoljene halucinarnom gratifikacijom;izmiču neumoljivoj nužnosti potrebe za prilagođavanjem na sredinu;objekt sex. pulzije se pojavljuje u potpuno posebnom položaju u odnosu na druge objekte.

12. Koji stadijum psihoanalize odgovara objektnom stadijumu kod Špica → Oralni stadijum

13. Karakteristike analnog karakteraUrednost - tačnost: ne samo u odevanju, već i u radu, načinu mišljenja, moralu, odnosu prema drugima.

Posvojnost Poštovanje pravila ponašanjaŠkrtost Dvojnost osećanja i težnji...Povišena osujetljivost

14. Šta je falusni primatFrojd je primetio da se u falusnoj fazi, gde se parcijalne pulzije pokoravaju investiranju genitalnih zona, jedino ceni muški polni organ i to isto tako kod dečaka, kao i devojčica. To je nazvao falusni primat.

15. Genitalni stadijum, razlika između ovom decomOko 2. godine dolazi do novih preokupacija kod deteta koje potvrđuju interes deteta za genitalnu zonu i za sve funkcije koje se njoj pridružuju.Dečija masturbacija svedoči da genitalni aparat dostiže status erogene zone.Ovu masturbacijutreba razlikovati od primarne masturbacione aktivnosti koja postoji već kod odojčeta i koja se ispoljava erekcijama, a me biti podržavana ručnim manipulacijama.Oko aktivnosti genitalne erogene zone organizuje se falusna faza. Psihički život deteta, njegove igre i radoznalosti, interesi i objektni odnosi polariziraće se buđenjem genitalne seksualnosti.

16. Indikacije za psihoanalizu po M. KlajnM. Klajn se ne obraća strukturnim datostima u pravom smislu. Ona pristupa ovom problemu

koristeći druge krterijume, treba otkritikako se nerešeni konflikti odražavaju na osobu, tj. proceniti patogenu nabijenost strahom i inhibirajući svojstva koja dečja neuroza vrši na psihičko f-sanje.

Njen dijagnostičiki postupak usmeren je na otkrivanje straha iza izvesnih simptoma, ali isto tako i na otkrivanje inhibicija nekih ponašanja.

Indikacije:Poremećaj sna,Teškoća ishrane,Fobični simptomi.

M. Klajn u prvom redu razmatra znake dečjeg straha, insistirajući na poremećajima sna, teškoćama ishrane i fobičnim simptomima. Poremećaj sna → uznemireno, često prekidano uspavljivanje, rana buđenja ili nesposobnost da se spava u toku dana. Ove teškoće su ekvivalenti noćnih strahova i direktno su vezani za fantazme koji navode na strah. Poremećaji u području ishrane mogu biti pokazatelji neuroze.

Originalnost M. Klajn → teži da proceni teškoće deteta, ne polazeći od simptoma, nego od promene svakodnevnih ponašanja. Ona privlači pažnju na izvesne inhibicije koje se odnose na aktivnost igranja.

43

Page 44: NEVENKA TADIC PDM - skripta

Indikacije će biti šire i pod „idealnim okolnostima“. M. Klajn ne vidi nikakvu prepreku za profilaktičnu

analizu sve dece. S praktičnog gledišta psihoanaliza će biti namenjena slučajevima gde dečja neuroza ne može naći spontano rešenje.

17. Tip „kao da“ na granici neuroze i psihozeVrsta shizoidnog karaktera, koji se ponaša kao da ima uobičajene emocionalne odgovore na situacije.

18. Šta je genetički pristup i kakav je kod poremećaja deceHartman i Kris su definisali genetičko gledište i suprotstavili ga dinamičkom. Genetički

pristup ide za tim da rasvetli ponašanje pozivajući se na istorijsko poreklo ili na anamnestičke podatke. Genetičko gledište dokazuje da je svaki psihički fenomen podvrgnut zakonima kauzalnostiu i da istoriju uzroka treba slediti do njihovog porekla, dalo je podstrek tzv. metodama posmatranja.

U svojim radovima, Genetičko gledište je akcenat na odnosu majka-dete i ističu prirodu objektivnog odnosa i posebno ispituju uticaj koji su na doživlje. i ličnost deteta mogli vršiti izvesni tipovi materinskog ponašanja, čak i izvesni društveni i kulturni uslovi.

19. Na kom stadijumu dete prelazi iz aktivnog u pasivni psihosexualni razvoj Na sado-analnom stadijumu

20. Kada se dete igra i u igri oživljava sukobe taj mehanizam odbrane je:Prisila ponavljanja.

21. Stadijum ogledalaJ. Lancan - stadijum ogledala - predstavlja važan trenutak u stvaranju jedinstvene slike o sebi i u doživljavanju sebe kao celine. Svoju sliku u ogledalu dete doživljava i prepoznaje kao celinu koja više nije “onaj drugi”.Doživljavajući svoje telo kao celinu, dete se oslobađa mučnih doživljavanja svog tela kao iscepkanog, rasparčanog i stopljenog sa drugima, iako su drugi i te kako prisutni u njemu i ostaju tamo do kraja života.Sve do 7. godine slika se pretežno odnosi na oblik tela - figurativna, a zatim postaje slika tela koje dejstvuje - operativna, što se može videti i na dečijem crtežu.

22. Rano afektivno lišavanjePotrebe odojčeta:1) Ppotrebe prvog detinjstva, potreba sisanja, potreba za stalnošću.2) Posledice ranog afektivnog lišavanja jedan od bitnih uslova, koji obezbeđuje mentalno zdravlje

osobe jeste uspostavljivanje osećajnog, trajnog odnosa između majke i deteta, odnosa koji daje sigurnost.

Ovaj odnos je neophodan da bi se kontrolisali strah i krivica, koji ne treba da prevaziđu normalne psihološke odbrane. Kad takav odnos nedostaje u prvom detinjstvu, govori se o ranom osećajnom lišavanju.

23. Razlike između neuroza i poremećaja nedostatakaRakamije razlikuje jasno „patologiju nedostatka“ i neurozu.Lišavanje uvodi u život početni nedostatak iz koga osoba mora da izađe oslabljena. Ni u jednom momentu nije reč o konfliktnoj frustraciji želja koju bi bilo moguće nadvladati i čije bi rešenje moglo sačinjavati, kao u edipovom komplexu, iskustvo koje oblikuje.

44

Page 45: NEVENKA TADIC PDM - skripta

Frustracija koja se sreće kod neurotičara nikada nema autentičnost ovog stvarnog nedostatka. Odnos sa roditeljima, koji je očuvan u neurozi, dozvoljava osobi da uđe u imaginarnu igru identifikacija, a iz koje je, afektivno lišeno dete upravo isključeno.

24. Načini reagovanja dece na odbijanje od dojke po M. RiblDva tipa:

1. Reakcija obeležena „negativizmom“ sa odbijanjem grudi, gubitkom apetita, mišićnom krutošću, površnim disanjem

2. Nazvan „depresivni tip“ označen je ravnodušnošću, mišićnom mlitavošću, bledilom i stomačno-cravnim poremećajima

25. Karakteristike oralnog karakteraKarl Abraham – zapazio uticaj oralnih iskustava na formiranje karaktera.

Opisao oralni karakter – podvlačio da oralno zadovoljstvo ne iščezava nikada potpuno i da libidoizne gratifikacije vezane za sisanje ostaju u nejednakim oblicima ponašanja odraslog.

Sreću se ekstremne varijacije od ličnosti obeleženih: optimizmom, poverenjem i ljubaznošću do ispoljavanja: zavisnosti, zavisti, neprijateljstva, uznemirenosti prema doživljaju prvih oralnih iskustava.

Obeležja koja se povezuju sa oralnošću:

Pasivnost, samoljubivost, agresivnost, pesimizam i optimizam, zavisnost, slab / dobar apetit, pohlepnost, sklonost ka griženju, pušenju, grickanju noktiju.

Ispoljavanje ovih crta ličnosti zavisiće od stepena učvršćenja instinktivne kateksije na objekt, u prvom redu libidne i agresivne.

Ako učvršćenje zahvata veliki deo energije, ostaje manje energije za prelazak u sledeće faze razvoja i za nove raspodele.

26. A. Frojd: problem transfera kod psihoanalize detetaOna smatra neophodnim da dete uspostavi srdačne odnose sa terapeutom, tj. „pozitivan transfer“.Uviđa da su odnosi između deteta i terapeuta afektivno nabijeni ljubavlju ili privrženošću, pobunom ili suprotstavljanjem. Ona kaže da ova osećanja ne sačinjavaju pravu neurozu transfera.Ako u toku analize odrasla osoba zaista može da dezinvestira objekte sa kojima je povezuju fantazmi i spremna je da ponovo stvori svoju neurozu u analitičkoj situaciji gradeći je oko osobe analitičara, takav transfer izgleda nemoguć u slučaju deteta, i to iz 2 razloga:

1) Za dete, početna situacija – odnos s roditeljima – ostaje uvek aktuelna, jer ono zavisi od njih. Ovi konfliktni objekti su spoljašnji i još nisu interiorizovani kao kod odraslog. Analitičar može da uđe u ovo kružno kretanje, ali on zaista ne može ni u kom slučaju da zauzme mesto roditelja, jer dete ne oseća nikakvu potrebu da u njemu nađe substitutivnu figuru.

2) Stav koji analitičar zauzima prema detetu je druga prepreka. Analitičar dece, po A. Frojd, treba da bude sve drugo nego „senka“, jer ne bi smeo da se odrekne svoje vaspitne uloge i dete mora da prepozna „normativni“ cilj terapijskog poduhvata → u ovim uslovima Ani Frojd analitičar izgleda kao osrednji transferni objekt.

Ona smatra da u normalnim uslovima transfer nije ni moguć ni poželjan. Smatra da se nikad ne radi o pravoj transfernoj neurozi, i to zbog spoljašnjih uslova, tj. stvarnog prisustva roditelja odgovornih za vaspitanje deteta.

45

Page 46: NEVENKA TADIC PDM - skripta

27. Razlike M. Klajn i A.Frojd u terapiji igromM. KlajnObratila se igrama i crtežima da bi uspostavila kontakt sa detetom i tako mu dozvolila da izrazi svoje fantazije. Igra, kao prirodna aktivnost deteta, učinila joj se kao idealan način, utoliko pre što se kroz igru, nastavljenu u prisustvu analitičara, izražava fantazmatski život. S jedne strane, igra izražava nesvesne sexualne i agresivne fantazme, ali u obliku igranja koje je istovremeno simboličko i aktivno. S druge strane, aktivnosts igranja u prisustvu analitičara daje igri novo značenje, pošto je analitičar tu da pokaže da „igra ide mnogo dalje od igre“ i da se on mešau detetov odnos sa libidinoznim objektima. Oko igre se uspostavlja odnos analitičar – dete, koji žigoše transferna situacija. Dete se igra u prisustvu analitičara i njegova igra tako gubi svoju spontanost. Ono se igra, ali se igra u situaciji pred nekim ko ga nadgleda i ko mu govori. Tako igra više nije spontana.A. FrojdNe zadovoljava se jedino verbalnim materijalom obraća se crtežima koje dete pravi u toku seansi. Ona ne poriče da bi se igra mogla interpretirati simboličkim izrazima, ali ona podvlači da – ako asocijacije koje stvara odrasli svedoče o voljnom naporu da se svesno ne prećuti ništa iz svog govora – igra deteta ne odgovara takvom stavu – ona naročito osporava mogućnost da se interpretira materijal igre u okviru transferne situacije, i ova teškoća za nju predstavlja najveću prepreku u analizi deteta.

28. Doživljaj prasceneFantazam koji potiče od Frojda.Predstavlja dramatizaciju edipovske situacije, u kojoj je Frojd prepoznao početak pravog sećanja.Sporno je da li postoji prvobitni fantazam o primarnoj sceni bez prisustva polnom činu između roditelja, ili dete svoje misli o polnom sjedinjenju između roditelja razvija nakon gledanja ovog odnosa.Dete može polni odnos između roditelja da shvati kao agresivni ili sadistički čin na osnovu prividnog izgleda sile koji ovde primenjuje jaći nad slabijim, otac nad majkom.Ovakva shvatanja mogu da budu praćena strahom od heteroseksualnih dodira u odraslom dobu.

29. Razlike u shvatanju transferne neuroze po A. Frojd i M. KlajnM. Klajn konstatuje da simptomi variraju zavisno od variranja od analize i od neuroze transfera.A. Frojd smatra da nema prave neuroze transfera kod deteta u toku koje analitičar u afektivitetu malog pacijenta zamenjuje prvobitne objekte. Za nju je svakodnevna okolina deteta, naročito njegovi roditelji, uključena u polje lečenja.Za M. Klajn stvarna okolina zauzima samo podređeni položaj u vezi sa unutrašnjim subjektivnim konflliktom. Za analizu vrede jedino projekcije i introjekcije dobrih i loših objekata. Transfer ima svoje poreklo u procesima koji u najranijim stadijumima određuju odnose s objektima.

30. Engleska škola - tehnike interpretacije interpretacije su date u ranom stadijumu analize, često od prvih seansi. interpretacije se odnose na nesvesni sadržaj igara i crteža, ciljanju u njima edipovske teme,

introjekcije i projekcije, sukob između spoljašnjih i unutrašnjih objekta, zavisno od primitivne scene i strahovanja od kastracije i komadanja.

na kraju se insistira na potrebi sistematskog interpretiranja negativnog transvera bez čega nijedna analiza ne bi mogla da bude dovedena do kraja i rizikovala bi da utone u imaginarnu igru s terapeutom.

46

Page 47: NEVENKA TADIC PDM - skripta

31. Malinovski – Edipov kompleksMalinovski insistira na kulturnom relativizmu osnovnih kompleksa oslanjajući se na proučavanje Melanezijskog stanovišta Trobrijanskih ostrva,Malinovski smatra da edipov kompleks može postojati samo u društvu patrijahalnog tipa, dok je na ovim ostrvima, gde se porodica struktuira preko matrilinearnog tipa, očev autoritet prenet na ujaka,u takvoj društvenoj strukturi edipo kompleks je premešten u kompleksnu matrilinearnu situaciju.

32. Indikacije za psihodinamsku psihologijuPotrebna je strukturna dijagnoza, koja treba da govori o neurotičnom trpljenju, bilo da osoba sama dolazi da trazi pomoć pod pritiskom neizdrživog straha, bilo da karakterni ili funkcionalni simptom remeti njeno društveno funkcionisanje, porodični život, kapacitete za ljubav ili rad.Dijagnozu postavljamo na osnovu uporedivanja izvesnog br. činjenica dobijenih sledećim izražavanjem:1) Anamnestičko ispitivanje, 2) Kliničko ispitivanje.

33. Zašto funkcionalni poremećaji nisu dovoljni da se dijagnostifikujuFunkcionalni poremećaj je reaktivni poremećaj ponašanja.Pojavljivanje fobičnih, opsesivnih simptoma kod deteta, školskih teškoća, poremećaja,

porodičnog ponašanja, psihomotornih ispoljavanja nije dovoljno da se potvrdi neuroza čiju dijagnozu treba tačno dokazati.

Uslovi sredine obično iščezavaju kada se poprave odgovorni spoljašnji uzroci, dok osoba još nije interiorizovala konflikt.

34. Kako M. Klajn opisuje težinu neurozeMerila:

1) Inhibicija epistemofilnih tendencija;2) Potiskivanje imaginarnog života;3) nesposobnost da se podnese frustracija;4) prekomerno prilagođavanje na vaspitne zahteve5) način afektivnih odnosa deteta sa njegovom porodičnom okolinom i drugom decom.

35. Melani Klajn – vreme javljanja neurozaDelovi ličnosti i meh. odb. su veoma ranog porekla, iz prvih meseci života i već na tom uzrastu postoji unutrašnji sukob (kako u okviru ono, između nagona života i smrti tako i između ono, ja i nad-ja) koji je gotovo nezavistan od okoline.

36. Uzroci mucanja kod deceMucanje je poremećaj govora. Poremećaj ritma, odnosno vremenskog odnosa između glasova u reči i rečenici. Takav govor karakteriše često ponavljanje ili produživanje glasova, slogova ili reči ili naizmenično česte pauze.Početak mucanje – obično pre 10. godine (između 3. i 5. g.).1 % dece nastavlja da muca i u adolescenciji → reakcije na događaje koji su stresni, burni.

Uzroci: ≈ Psihogeni ≈ Organski i ≈ Fiziološki

Pored nedovoljnog biološkog sazrevanja, zona mozga odgovornih za govor, važnu i procentualno znatno češću ulogu u nastanku poremećaja imaju emocionalni faktori (stres u detinjstvu) i uticaj okoline u kojoj dete živi (poremećen odnos dete-roditelji).Mucanje je najčešće traumatogenog porekla. Jedan od uzroka je strah od sopstvene agresije, pa reč zastaje u grlu.Češći kod dečaka.

47

Page 48: NEVENKA TADIC PDM - skripta

37. Ciljevi terapije roditeljaTerapeut neće moći da deluje ako smenjuje autoritativnog, omrznutog ili željenog roditelja, ili ako preuzima ulogu „zamene roditelja“.Njegova akcija postaje uspešna samo pod uslovom da pomogne roditeljima da postanu svesni psihičkih konflikata koji napinju njihov odnos sa detetom i da ih reše.

38.Šta je pripremna faza, primarni proces i obeležja Pripremna faza

A. Frojd predlaže da svakoj psihoanalizi deteta prethodi pripremna faza. Smisao ove faze je da se od samog početka sredi nepogodna situacija, tj. da se dete motiviše da započne i izdrži terapijski odnos. U toku nje osoba sa terapeutom može da uspostavi veze zasnovane na poverenju. Terapeut treba detetu da se učini kao osoba koja se interesuje za njega, koja mu može biti korisna i koja raspolaže izvesnom moći. Terapeut treba da postane neophodan detetu.

Primarni proces:Prvobitni način nesvesne misaone delatnosti u kome psihička E slobodno, bezvremeno protiče od jedne predstave do druge, premešta se i zgušnjava.

39. Psihoterapija podrške kod roditeljaTeži da:

1) umanji frustraciju i osećanje krivice,2) podigne granicu tolerancije prema psihoterapiji njihovog deteta,3) rasvetljava značenje koje ova terapije ima za dete, kao i razočaranja i teškoće koje ona

povlači,4) ova terapija podrazumeva razgovore koji odražavaju, u promenljivom ritmu, trajni kontakt s

jednim ili oba roditelja.

Psihoterapija podrške:Reč je o tome da ona ne kani da direktno razreši neurotične probleme roditelja, već da olakša napetosti prouzrokovane psihoterapijom deteta.Tehnike u okviru ove terapije:CASE-WORK → omogućava ne samo da se uskladi ova psihoterapija sa roditeljskim reakcijama, već često da se ide izvan toga da bi se odrasla osoba suočila sa svojim vlastitim problemima.

40. Razlika između neurotičnih i psihotičnih mehanizama odbrane po M.KlajnPsihotični mehanizmi odbrane, tj. cepanje, napušteni i zamenjeni „neurotičnim“ odbranama, potiskivanjem, premeštanjem i inhibicijom.Ona smatra da paranoidne i depresivne pozicije predstavljaju tačke psihotične fiksacije. Kad osoba regradira na one arhaične faze svog razvoja, ona gubi značenje realnosti i postaje psihotična. Ako je osoba bila sposobna da prevaziđe ove stadijume, neće regradirati na taj nivo i moći će postati neurotična...Ona vidi razliku između psihoze i neuroze uglavnom u dubini regresije: neurotična regresija nedostiže nikad prve faze...

41. Dečiji crtežAna Frojd se ne zadovoljava jedino verbalnim materijalom i takođe se obraća crtežima koje deca prave u toku seansi. Često se događa da crteži zamenjuju svaki drugi način komunikacije između deteta i terapeuta. Dete se može obratiti crtežu da na svoj način predstavi stvarne ili imaginarne scene. Priče koje osim latentnog značenja koje se ocrtava iza ovih jasnih produkcija, moći će da služi kao mamac slobodnim asocijacijama.

48

Page 49: NEVENKA TADIC PDM - skripta

PSIHOLOGIJA JA

1. Dinamičko, ekonomsko i topičko gledišteDinamičko – sve motivacije ljudskog ponašanja su konfliktne i premeštanje od principa

zadovoljstva ka principu realnosti predstavlja jedan od vidova prilagođavanja čija dinamika oscilira između prevage direktnog zadovoljenja pulzija i osećanja vitalne sigurnosti ili bolje rečeno provage ono i ja.

Ekonomsko – pristupa psihičkim fenomenima sa kvantitativnog stanovišta prisutnih snaga. Problemi su postavljeni u izrazima pulzione energije osećajnog investiranja.

49

Page 50: NEVENKA TADIC PDM - skripta

Topičko – posmatra probleme strukture psihičkog aparata. Razmatra da se suprotstavljanje svesno-nesvesno, odnos između istanca ličnosti zbog uloge koju ove igraju u konfliktu i načinu na koji posreduju u formiranju ličnosti.

Jednom pojmu, kao potiskivanju, se može pristupati sa gledišta topike, ali on otkriva i probleme dinamike i ekonomike.

2. Nad-ja kod muškaraca i ženaKod muškarca edipov kompleks nije bio jednostavno potisnut, nego se rasprsnuo pod udarom

pretnje kastracije. Njegova libidonozna investiranja su napuštena, lišena seksualnosti, sublimirana s jedne strane njegovi objekti su inkorporirani u ja gde obrazuju jezgro nad-ja. U idealnom slučaju edipov kompleks ne ostaje da živi čak ni u nesvesnom, budući da je nad-japreuzeo njegovo nasleđe.

Kod žena zbog toga što ona može malo po malo da napusti edipov kompleks i da ga reši potiskivanje, posledice će biti sasvim različite. Nad-ja žene neće nikad biti tako neumoljiv, bezličan, odvojen od svih osećajnih potreba kao kod muškarca. U posebnosti izgrađivanja ženskog nad-ja Frojd vidi poreklo izvesnih crta svojstvenih ženi: manju potčinjenost velikim nevoljama života. Njene odlike ostaju podređene nežnim i neprijateljskim osećanjima, ona je manje osetljiva na pojmove pravde i pravo.

3. Frojd - jaJa se diferencira u psihičkom aparatu u kontaktu sa spolj. realnošću. Njegovo funkcionisanje je upravljeno sekundarnim procesima. Njegova struktura je određena principom realnosti.ja

1) Kontroliše voljni modalitet - f-je konzervacije2) Opaža stimulise - mnestička f-ja3) Registruje sećanja - mnestička f-ja4) Prilagođava se draženjima i uzbuđenjima - f-ja prilagođavanja5) Izbegava suviše vezive napetosti - f-ja izbegavanja6) 6.proučava obaveštenja koja mu pristižu i teži da izmeni sredinu stvarajući je podobnijom

da odgovori na njegove potrebe i da ispuni njegove želje - f-ja učenjameh.odbrane - frojdHisterična neuroza - potiskivanjeOpsesivna neuroza - izolacija i reaktivna formacija

4. Frojd – razlika tuge od depresijeOdbrana od depresije obavlja se kroz proces i rad tuge, žalosti u nekoliko etapa. Pod procesom tuge se podrazumevaju oni obmanjujući mehanizmi koji se pokreću sa ciljem uspostavljanja ravnoteže ja u stanjima depresije. Mehanizmi tuge ili odbrane tugom rezultat su rasta i razvoja psihološkog aparata koji ima načine da obradi depresiju nastalu na gubitak objekta i zavisiće od faze razvoja u kome se iskustvo deprasije javlja.

5. Frojdovo ispitivanje – nali HansOn neurozu odraslih dovodi u vezu sa dečijom neurozom. Poziva se na E. komplex i kastracioni strah kada govori o jezgrovnom konfliktu koji strukturira neuroze i koji je mogao biti ustanovljen u jednom kliničkom slučaju, opservacija prve „analize“ deteta, analize malog Hansa.Ova fobična neuroza kod deteta od 5 godina koju je Frojd analizirao oko 1906. omogućila mu je da prouči teorije o dečjoj sexualnostij, kako ju je on rekonstruisao, počevši od materijala sakupljenog u toku analiza odraslih.

50

Page 51: NEVENKA TADIC PDM - skripta

6. Telesno ja u snovima po Federnuosećanje telesnog ja – složeno i uključuje sva motorna i senzorna sećanja koja se tiču same osobe.može da se smatra delom osećanja mentalnog ja. Kod naglog padanja u san osećanje telesnog ja nastaje pre osećanja mentalnog ja.telesno ja može sasvim nestati tokom padanja u san i može biti ponovo investirano i probuđeno od strane mentalnog ja, koje ostaje budno. Na taj način možemo uspeti u dobrovoljnom odlaganja spavanja.kod normalnog procesa buđenja telesno i mentalno ja se bude gotovo istovremeno.u snovima sa karakterističnim senzacijama – bolnim, kao neprijatnim – telesno osećanje ja je uvek pojačano, ali često nepotpuno.ovo je osećanje i naglašeno i potpuno u snovima letenja i plivanja, koji su praćeni istaknutim osećanjem dobrobiti.telesno osećanje ja nije identično sa telesnom šemom, može da bude uže ili šire od nje, zavisno od kvaliteta ili kvantiteta katektiranosti, odnosno uloženosti ja.telesno osećanje ja uključuje: motorna i čulna osećanja u vezi sa sopstvenom ličnošću.ja prelazi iz jednog stanja u drugo ali uvek uz doživljaj jedinstva i trajanja koji se tokom privremenih prekida tokom sna ponovo uspostavlja.

7. Federn: derealizacija – depersonalizacijaGranica ja predstavlja periferni čulni organ koji stoji između ja i ne-ja, a čija katektiranost određuje zdravlje ja tj. ličnosti, omogućava razlikovanje realnog i nerealnog. Kada spolj. granica izgubi svoju kateksu spoljašnji objekti se doživljavaju kao strani, nepoznati – fenomen derealizacije.Slabljenje unutrašnje granice dovodi do halucinacija koje se doživljavaju kao realne jer potiču od mentalnih stimulusa koji su ušli u svest a da nisu uložene, invastirane od ja – fenomen depersonalizacije.Slabosti ja je posledica nedostatka izvora kateksi od kojih libido igra najznačajniju ulogu. Oštećenje ja i njegovih funkcija nisu uslovljeni pojačavanjem uloženosti ja već slabljenjem tj. lezije granice ja.

8. Kakvo je telesno JA u snovimaPojam telesnog JA je tesno vezan za JA, pod kojim je Frojd podrazumevao ovaj deo JA koji potiče od sopstvenih samooperacija o sebi, nasuprot percepcija o spoljašnjim objektima. Telesna šema je neurološki termin za organske reprezentacije tela u mozgu, kok je slika o sebi psihološki termin za shvatanje koje Self ima o vlastitom telu.

9. Kako se određuje snaga egaObraćajući se snazi ja, kvantitativnoj datosti, Ana Frojd razmatra uspešnost ja u njegovom

zadatku da se odupre instiktivnim pulzijama. Ja treba da se ponaša kao istanca koja reguliše želju. Nakon „neposredne vladavine instinkta“ dete stiče mogućnost da radi ostvarivanja svojih želja koristi spoljašnji svet. Identifikacijom sa roditeljima ja osobe pokušava da upravlja svojim željama i pred pulzijama prihvati kritični stav koji odgovara interiorizovanim roditeljskim zahtevima. Princip zadovoljstva malo po malo zamenjuje princip realnosti koji teži da izmiri želje sa zahtevima iz spoljašnjosti. Ova funkcija posrednika pada na ja koja se tako nalazi u konfuznoj poziciji sa ono. Ana Frojd u uspostavljanju ravnoteže između pulzija ono i snaga ja vidi bitan uslov za harmoničan razvoj ličnosti.

51

Page 52: NEVENKA TADIC PDM - skripta

10. Kada se javljaju funkcije ega- HartmahOno i ja nastaju kao nezavisni sklopovi iz prvenstveno nediferencirane osnove (biofiziološke

matrice). Ja se ne razvija isključivo učenjem i iskustva, već je delimično autonomno i poseduje svoje biološke korene u urođenim procesima koji su analogni fiziološkim procesima.

Samim tim, nisu sve funkcije ega posledica sukoba instinktivnih pokretača sa stvarnošću, već f-je ega kao npr. pamćenje, opažanje itd. postoje pre sukoba i nezavisne su od njega, odnosno urođene su i nekonfliktne.

Iz prvobitno neizdiferenciranog stanja ne razvija se samo ono već i autonomno ja sa svojim funkcijama primarne autonomije. Vremenom ove aktivnosti mogu da budu uključene u sistem ono odnosno da budu u službi ono-g ali i u službi nad-ja.

Pored primarne postoji i sekundarna autonomija, pošto oblici ponašanja mogu od sredstva da potanu cilj za sebe. Svaka navika koja se stiče kao odbrana ja može da razvije svoju sekundarnu autonomiju koja obezbeđuje sebi opstanak i onda kada potreba za odbranom više ne postoji (preterana urednost koja ostaje i nakon nestanka strepnje koja ju je uslovila). Reč je o odvajanju jednog procesa koji se razvija u službi instinktivnog pokretača i njegovog funkcionisanja kao da je sam sebi cilj.

Dolazi do promene u funkciji. Ovaj pojam obezbeđuje razumevanje odbrana, prilagođavanja i efikasnosti ja

11. Razlika Ideal JA i Idealno JAJA ideal je idealna predstava o samom sebi kojoj se teži, to je jedna od f-ja Super Ega pa se ponekada koristi kao sinonim za njega. JA idealom se ističe razlika: konflikt sa Super Egom rađa krivicu, sa JA idealnom radnjom stid. Idealno JA je idealizovana pounutrena slika roditelja, nastala primarno iz Ega i primarno je psihička tvorevina.

12. Sličnost i razlike po WalderuZajedničke crte mehanizama odbrane:Oni teže da iz svesti isčeznu svi mučni afekti ili afekti koji se često zadovoljavaju.Oni su često specifični za određenu osobu tj. pronalaze se uvek isti kod iste osobe pre nego noki drugi.Meh. odb. dovode do pojave substitutivnih gratifikacija, koje su neprilagođene svom prvobitnom objektu, ili koje zauzimaju mesto u obliku neurotičnih simptoma, nemoguće gratifikacije.

13. Uloga bajki kod deceBajke su kod dece posebno prihvaćene na prelazu iz konkretnog na formalno-operacionalni period (oko 10. god.) jer odgovara razvoju saznajnih funkcija.Bajke pomažu razrešavanju mnogih emocionalnih problema kod dece, posebno strahova.Nesvesni sadržaji, koji se pojavljuju u snovima, u bajkama bivaju jasnije operacionalisani i svesno doživljeni, obično sa srećnim i povoljnim ishodom.Psihoanalitičko tumačenje bajki je blisko tumačenju mitova i snova. One se takođe shvataju kao rad nesvesnog kroz određene simbole i njihove relacije.

14. Šta je aleksitimija? Nesposobnost da se protumače i na simboličan način izrazeosećanja.

15. Strah od odvajanja - gde se javljakod simbiotske psihoze, kod školske fobije, oživljava se i ponavlja strah od odvajanja,može kod slepih i gluvih zato što su previše zavisni od drugih.

52

Page 53: NEVENKA TADIC PDM - skripta

simbiotička psihoza poremećaji uspavljivanjaanaklitička depresija bronhijalna astmaškolska fobija kod gluvih i slepih

16. Deca prevazilaze strah kroz?Umereno zastrašujuće prilike, osobe, igre, priče koje ih uzbuđuju, umereno plaše i podstiču na

maštu, predviđenje i delatnosti.

17. Deca ponavljanjem prevazilazeSveža povređujuća i osujećujuća iskustva, pokazuju težnju da se ponavljaju krooz snove. Kroz ponavljanje povrede u snu psihološki aparat uči kako da ovlada strepnjom koju povreda izaziva. Neprijatna snovna iskustva mogu oživeti stara potisnute sukobe.

18. Odgovori dece na razvodpod najpovoljnijim okolnostima agresijom, poricanjem gubitka, udaljavanjem od roditelja i strastveno predavanje sportu, muzici, učenju. Koristi situaciju da dobije novac, izlaske, opravdanja, neuspeh u školi.

- mlađa deca – smetnje apetita, spavanja, deficiranja, bes, nemir- starija deca – osećajna hladnoća, strepnja i fobije, osećaj krivice, depresija, pokušaji

samoubistvapod nepovoljnim okolnostima (najnepovoljnijim) deca i omladina koji su ispoljavali smetnje prilagođavanja mogu ispoljavati neurotične i psihotične odgovore i stanja, ponašanja, psihosomatska oboljenja. Ponekad je bolji dobar razvod nego loš brak.

19. Strah od smrtiDete se prvi put sa smrću susreće u susretu sa mrtvim životinjama. Brani se povlačenjem,

udaljavanjem i privijanjem uz odraslu osobu. Ovi odgovori se javljaju pre nego što dete ima svest o smrti. Dete mlađe od 3-4 godina nema izgrađen pojam o smrti, shvata ga kao odlazak umrle osobe iz kuće, 7-8 godina postoji kao činjenica, ali se još uvek ne uviđa mogućnost vlastite smrti, 8-9 godina shvataju da su oni smrtni. Javlj se naćna mora, strah, strah od odvajanja od stvarnosti.

20. Kognitivno-bihejvioralna terapija kod depresijePrema teoriji učenja depresija je izazvana nedostatkom potkrepljenja i prisustvom averzivnih

stimulusa. Ako je izgubljen važan objekat koji je nadzirao veliki broj pokušaja deteta, gubi se potkrepljivač i ova ponašanja se prekidaju. S druge strane, bezpomoćnost je naučen stav o životu koji je tesno povezan sa depresijom. Agresija je smanjena kod depresivnih po teoriji učenja.

21. Kožna osetljivostMokrenje u postelju i rublje može biti izraz težnji ka pasivnosti i za kožnim zadovoljenjem kod mlade dece.

22. Deca koja nisu pogodna za psihoterapijuNisu formirane istance ja i nad-jaPoremećaji ponašanja (hiperkinetički sindrom)Umna zaostalost (sa psihotičnom dimenzijom)Depresivna (mutizam)

23. Odbačena i preterano zaštićena decaPreterano su usmereni na sebe Bez preduzimljivostiImaju teškoće u odnosu sa drugima Imaju osećaj manje vrednostiPasivni su Zavisni su

53

Page 54: NEVENKA TADIC PDM - skripta

24. Neuroza sukoba – je po Ani Frojd u edipalnoj a po Melani Klajn u pre-edipalnoj fazi

25. Koji mehanizam deca najčešće koriste u svojim igrama- ponavljanje

26. U koju vrastu poremećaja spada neosetljivost kože i sluzokožeporemećaj hranjenja

27. Preobjektni stadijum po Frojdu naziva sestadijum primarnog narcizma

28. Razlike između mehanizama odbrane i strategije prevladavanjaMeh. odbrane su crta ličnosti po bihejvioristima, aStrategije prevladavanja su bihejvioralne i izaziva ih stres.

29. U kom periodu su fiksirane osobe sa graničnim smetnjamau ranom uzrastu

30. Dali adolescenti koriste projektivnu identifikacijuDa, stvaraju idole.

31. Analizom perioda separacije bavila seMargaret Maler (16 meseci do 3 god.)

32. O kom odbrambenom meh. je reč kad se stalno ponavljaju priče decirepeticija (prisila ponavljanja)

33. Šta su DSM i DMDSM – dijagnostičko-statističko merilo,DM – dijagnostičko merilo

34. Najpoznatije Hartmanovo delo –“Ego psihologija i problemi adaptacije”

35. Navedite bar jednog autora koji se zalaže za koncept strategije prevladavanjaLazarus i Mus

36. Dečiji crtežAna Frojd se ne zadovoljava jedino verbalnim materijalom i takođe se obraća crtežima koje deca prave u toku seansi. Često se događa da crteži zamenjuju svaki drugi način komunikacije između deteta i terapeuta. Dete se može obratiti crtežu da na svoj način predstavi stvarne ili imaginarne scene. Priče koje osim latentnog značenja koje se ocrtava iza ovih jasnih produkcija, moći će da služi kao mamac slobodnim asocijacijama.

54

Page 55: NEVENKA TADIC PDM - skripta

37. Kako F. Dambar tumači psihosomatske poremećajeUočavaju se zajedničke psihološke osobine psihosom. ličnosti bez obzira na vrstu poremećaja. Prva osobina - nesposobnost bolesnika da uoče psihogenu pozadinu svojih telesnih zbivanja. Druga - snižen kapacitet simboličkog izražavanja

skloni su tehničkom rešavanju problema uz smanjenu sposobnost za umetnički doživljaj imaju potrebu za redom, radom i tačnošću simbiotski životni stil simbioze su često ambivalentne, ali ih oni ne znaju, niti mogu prekinuti, a ako ih drugi

prekinu, moguće su psihotične dezintegracije šizofrenog tipa, depresivnog ili maničnog, toksikomanije ili suicidnost.

svoje konflikte izražavaju u okviru socijalno dozvoljenog, jer samim prihvatanjem uloge bolesnika kroz psihosom. Simptome izbegavaju asocijalnost.

38. Indikacije za roditeljsku psihoterapiju Indikacije izmiču krutoj sistematizacije. U takvoj perspektivi dve su mogućnosti :

A. Da se savetuje roditeljima, ili jednom od njih, da započnu analitičku psihoterapiju koja bi težila da razreši neurotično jezgro ukidajući nesvesne faktore koji održavaju psihički konflikt.

B. Da se započne psihoterapija podrške, uz nadanje da će izvesne promene stavova okončati osećanje interakcije koje uslovljavaju porodične teskoće.

Svaka psihoterapija teži da postigne osećajno sazrevanje, u onom smislu u kome se očekuje da isčeznu infantilna ponašanja koja se održavaju kod roditelja kao posledica starih nerešenih konflikata.

Svaka psihoterapija, ma kako bila površna, upliće ceo terapijski odnos u spor i teži da istakne ne samo objektivnu realnost, nego i sve to što je obuhvaćeno u imaginarni splet koji joj daje boju i simboličko značenje koje joj se pridaje.

39. Margaret Maler –201. strana

40. Setting u psihoterapijiTerapeut unapred propisuje kakva će terapija da bude, koliki će bit broj seansi, koliko će trajati, vrši pripremu deteta i roditelja, određuje prostoriju koja će uvek biti ista.

41. Individuacija...je proces postepenog ulaganja f-ja JA, u prvom redu stvaranja stalne predstave o selfu koja se oslanja na predstavu o objektu. Kroz odvajanje dete postepeno izranja iz simbiotičkog odnosa sa majkom i razlučuje se njegov Self od majčinog.

55